SlideShare una empresa de Scribd logo
1 de 35
Genodermatoses
1) Tyrosinase positive albinism (oculocutaneous albinism type 2) is caused by a mutation in which
of the following:

A. Tyrosinase
B. C-kit
C. NEMO
D. Tyrosinase related protein 1
E. P gene Correct Choice
Oculocutaneous albinism (OCA) type 1 (Tyrosinase negative albinism) is caused by mutations in the
tyrosinase gene. OCA type 2 (tyrosinase positive albinism) is caused by mutations in the P gene.
OCA type 3 is caused by mutations in the tyrosinase related protein 1 gene. C-kit mutations cause
piebaldism and NEMO mutations cause incontinentia pigmenti.


2) A patient with a port wine stain covering one enlarged leg likely has which of the following
associated symptoms?


A. Enchondromas

B. Visceromegaly with omphalocele

C. Distichiasis

D. Lymphatic and deep venous insufficiency Correct Choice

E. Bilateral retinal hemangioblastomas
Klippel-Trenaunay-Weber syndrome is a sporadic condition characterized by port-wine stains
typically covering one lower extremity that is enlarged with underlying lymphatic and deep venous
insufficiency.


3) Patients with Hermansky-Pudlak syndrome may experience which of the following systemic
complications as a result of their disease?


A. Aortic stenosis

B. Rectal abscesses

C. Arteriovenous malformations

D. Gastroesophageal reflux disease

E. Pulmomary fibrosisCorrect Choice
Patients with Hermansky-Pudlak syndrome are tyrosinase positive albinos. In addition to their
pigmentary dilution and increased risks for cutaneous malignancies, patients lack platelet granules
leading to impaired platelet aggregation (and therefore a bleeding diathesis). There is also a
lysosomal membrane defect which leads to the accumulation of ceroid lipofuscion in macrophages
within the lung (leading to pulmonary fibrosis), gastrointestinal tract (leading to granulomatous
colitis), and heart (cardiomyopathy).


4) Non-bullous icthyosiform erythroderma is caused by which of the following mutations

A. 12R-lipoxygenase gene (ALOX12B)

B. lipoxygenase-3 gene (ALOXE3)

C. All of these answers are correctCorrect Choice

D. transglutaminase-1 gene (TGM1)




                                                    1
E. Both 12R-lipoxygenase gene (ALOX12B) and lipoxygenase-3 gene (ALOXE3) are correct
Non-bullous congenital erythroderma (NCIE)is an autosomal recessive disorder characterized by a
collodion baby presentation at birth, and generalized erythroderma with fine white scale,
palmoplantar keratoderma, and heat intolerance. NCIE may be caused by mutations in
transglutaminase-1 gene (TGM1), the 12R-lipoxygenase gene (ALOX12B), and the lipoxygenase-3
gene (ALOXE3). Mutations in the keratinocyte TGM1 gene interferes with normal cross-linking of
structural proteins and the lipid envelope, leading to defective cornification and desquamation.
ALOXE3 functions as an epoxy alcohol synthase using the product of ALOX12B as the preferred
substrate; either gene can be the site of mutations causing NCIE.


5) In which of the following Genodermatoses would one find cutaneous hyperpigmentation, blue
lunulae and Kayser-Fleishcher rings


A. Hemochromatosis

B. Osteogenesis Imperfecta

C. Marfan's Disease

D. Gaucher's Disease

E. Wilson's DiseaseCorrect Choice
In Wilson's disease (Hepatolenticular Degeneration) one will find a vague greenish discoloration of
the skin on the face, neck, and gentalia Hyperpigmentation), azure lunulae (sky-blue moons) of the
nails, and Kayser-Fleischer rings. This is due to the body retaining excessive amounts of copper


6) Christ-Siemens-Touraine Syndrome is most commonly linked with defects in which of the
following genes?


A. ectodysplasin (EDA)Correct Choice

B. ERCC2

C. NEMO

D. None of these options are correct

E. ATP7A
Ectodysplasin (EDA) on Xq12-q13 is transmitted in an X-linked recessive fashion and is the most
common cause of anhidrotic ectodermal dysplasia (Christ-Siemens-Touraine syndrome or
hypohidrotic ectodermal dysplasia). NEMO can be linked to this syndrome and is associated with
immunodeficiency, but this is a rare association. ERCC2 is associated with trichothiodystrophy and
ATP7A with Menkes kinky hair syndrome. They are not associated with anhidrotic ectodermal
dysplasia.


7) Cronkhite-Canada is associated with gastrointestinal polyposis, nail atrophy, alopecia,
generalized pigmentation of skin and melanotic macules on the fingers. Which of the following
describes its inheritance pattern?


A. X-linked dominant

B. X-linked recessive

C. Autosomal recessive

D. Autosomal dominant

E. SporadicCorrect Choice
Cronkhite-Canada is associated with gastrointestinal polyposis, nail atrophy, alopecia, generalized
pigmentation of skin and melanotic macules on the fingers. It is inherited in a sporadic manner.



                                                  2
8) A biopsy is taken from a patient with multiple facial lesions is shown. They also have oral
papillomas and acral keratotic papules. Which of the following genes is most likely involved in the
etiology of these lesions?


A. CDKN2A

B. PTENCorrect Choice

C. NCF1

D. NCF2

E. CYBA
The biopsy shown is a trichilemmoma. Patients with Cowden syndrome (multiple hamartoma
syndrome) will have multiple facial trichilemmomas in addition to the other findings mentioned. Also
present are multiple other adenomas and potentially adenocarcinomas, especially of the thyroid and
breast. Hamartomatous polyps of the gastrointestinal tract are also present.

The other listed genes are not involved in Cowden syndrome. CYBA, NCF1/2 are involved in chroinc
granulomatous disease and CDKN2A is associated with familial dysplastic nevi.


9) Ectopia lentis (downward displacement of the lens) is characteristic of:

A. Multiple Carboxylase deficiency

B. HomocystinuriaCorrect Choice

C. Ehlers-Danlos syndrome

D. Phenylketonuria

E. Marfan syndrome
Ectopia lentis (downward displacement) is seen in homocystinuria. Upward displacement is seen in
Marfan syndrome. There are no changes in the lens in phenylketonuria or multiple carboxylase
deficiency


10) An infant presents with multiple congenital hemangomas in an generalized distribution. What is
the most serious associated condition?


A. None of the answers are correct

B. All of the answers are correct

C. Obstructive jaundice

D. Congestive Heart FailureCorrect Choice

E. Portal hypertension
High output congestive heart failure can lead to death in these children. Obstructive jaundice and
portal hypertension both occur, but are less likely to cause death. The hemangiomas will undergo
spontaneous regression.


11) What medication may exacerbate this autosomally dominant, acnatholytic disorder?

A. LithiumCorrect Choice

B. Oral contraceptives

C. Phenytoin



                                                  3
D. Corticosteroids

E. Anti-malarials
Darier's disease is autosomal dominant condition characterized by hyperkeratotic papules coalescing
into warty plaques and cobblestoned papules on mucosal surfaces. The cutaneous manifestations
may be exacerbated by lithium.


12) Which of the following subtypes of Ehlers-Danlos Syndrome (EDS) is associated with early
demise?


A. Type 4 (Vascular) Correct Choice

B. Type 5 (X-linked)

C. Type 1 (Gravis)

D. Type 7 (Arthrochalasis multiplex congenita)

E. Type 10 (Fibronectin)
Ehlers-Danlos Syndrome type 4 (Vascular) is caused by mutations in collagen 3 in some cases.
There is a tendency to develop arterial and visceral rupture resulting in early death.


13) An infant presents with poikiloderma on his face, buttocks, arms and legs. He is also noted to
have a hypoplastic thumb and no radius. Yearly ophthalmologic examination is indicated because of
the infant is at risk for developing:


A. Subcapsular lens displacement

B. Macular degeneration

C. Glaucoma

D. Copper deposition

E. CataractsCorrect Choice
The patient described has Rothmund-Thomson syndrome (or poikiloderma congentiale), an
autosomal recessive disease localized to chromosome 8 and believed to be due to a DNA helicase
mutation (RECQL4). 40-50% of patients will develop juvenile cataracts before puberty. Other clinical
features include alopecia, dystrophic nails, short stature, hypogonadism and dental dysplasia.


14) Retention of primary teeth a dental finding of which of the following conditions?

A. Letterer-Siwe disease

B. Tuberous sclerosis

C. Hypomelanosis of Ito

D. Hyper-IgE syndrome Correct Choice

E. Jackson Sertoli syndrome
Hyper-Immunoglobulin E syndrome is an autosomal dominant condition with impaired regulation of
IgE function and deficient neutrophil chemotaxis. There is increased susceptibilty to infections and
increased IgE serum levels. Retained primary teeth and lack of development of secondary teeth are
characteristic findings. The remaining conditions do not have this as a prominent finding.


15) A patient with a white, spongy overgrowth of the buccal mucosa that has passed in an
autosomal dominant fashion is most likely related to a mutation in which of the following?




                                                  4
A. Keratin 6b/17

B. Keratin 1/10

C. Keratin 6a/16

D. None of these options are correct

E. Keratin 4/13Correct Choice
This description is most likely a white sponge nevus, an autosomal dominant defect in keratin 4/13.
Keratin 1/10 is mutated in epidermolytic hyperkeratosis and Unna-Thost PPK, Keratin 6a/16 in
pachyonychia congenita type I, and keratin 6b/17 in pachyonychia congenita type II.


16) Dental enamel pits are seen in which of the following conditions?

A. Hypomelanosis of Ito

B. Tuberous sclerosisCorrect Choice

C. Hyper-IgE syndrome

D. Letterer-Siwe disease

E. Jackson Sertoli syndrome
Tuberous sclerosis is an autosomal dominant condition caused by mutations of the TSC1 (hamartin)
or TSC2 (tuberin) genes. These are tumor suppressor genes. Skin findings include hypopigmented
macules, connective tissue nevus, facial angiofibromas, periungual fibromas and cafe au lait
macules. Dental enamel pits and gingival fibromas are oral findings that are associated with this
condition. The remaining conditions do not have dental pits.


17) A 4-year old boy presents with generalized white scale. The mother reports that her son was
born with a tight membrane enveloping his body. Peripheral blood smear is within normal limits.
What is the most likely diagnosis?


A. ichthyosis vulgaris

B. lamellar ichthyosis

C. neutral lipid storage disease

D. netherton syndrome

E. Congenital ichthyosiform erythrodermaCorrect Choice
The most likely diagnosis is Congenital ichthyosiform erythroderma. In neutral lipid storage disease,
the peropheral blood smear would demonstrate lipid vacuoles in leukocytes and monocytes.
Lamellar ichthyosis is characterized by plate-like scale in children/adults. Netherton syndrome is
characterized by ichthyosis linearis circumflexa. Ichthyosis vulgaris does not typically present with
collodian baby


18) Mosaic mutations in PTEN are seen in which of the following conditions?

A. Gardner syndrome

B. Noonan syndrome

C. Incontinentia pigmenti

D. Proteus syndrome Correct Choice

E. Beckwith-Wiederman syndrome




                                                  5
Proteus syndrome is a sporadic condition caused by postzygotic mosaic mutations in PTEN. Clinical
features include subcutaneous lymphovenous malformations, capillary malformations, lipomas,
connective tissue nevi of palms/soles, hemihypertrophy, frontal bossing, hyperostoses of epiphyses
& skull (especially external auditory canal), scoliosis, bilateral ovarian cystadenomas, and parotid
monomorphic adenomas.


19) Which syndrome is characterized by hyperhidrosis, lack of pain sensation, hypersalivation, and
absent fungiform papillae?


A. Rubinstein-Taybi syndrome

B. Noonan Syndrome

C. Turner Syndrome

D. Riley-Day Correct Choice

E. Cornelia de lange Syndrome
Riley-Day syndrome is also known as Familial Dysautonomia. It is an autosomal recessive disorder
with the gene defect on the long arm of chromosome 9. Patients have unmyelinated sensory and
sympathetic neurons and autonomic dysfunction, leading to hyperhidrosis, decreased corneal
sensation and tear flow, hypersalivation, gastroesophageal reflux, decreased deep tendon reflexes,
and lack of pain sensation. They also exhibit abnormal histamine skin test.


20) Homocystinuria is caused by a defect in:

A. holocarboxylase synthetase

B. biotinidase

C. phenylalanine hydroxylase

D. cystathione beta-synthetaseCorrect Choice

E. gp91-phox
Cystathione beta-synthetase is defective in homocystinuria, an autosomal recessive conditions
characterized by increased homocystine and methionine levels in blood and urine. Other findings
include a malar flush, DVTs/emboli, cardiovascular disease, livedo reticularis, leg ulcers, blonde
hair/fiar complexion, downward lens dislocation, glaucoma, mental retardation, seizures, psychiatric
disorders and a marfanoid body habitus. The other enzymes are not involved in this condition.


21)
Hypoplasia of the breast can be seen in which
disease?


 A. anhidrotic ectodermal dysplasiaCorrect Choice

 B. osteogenesis imperfecta

 C. congenital syphilis

 D. Marfan syndrome

 E. Maffucci syndrome
Anhidrotic ectodermal dysplasia is a X-linked
recessive disease caused by mutations in
ectodysplasin, a member of the tumor necrosis
family. Patients may have dry skin with pigmentation
periorbitally, hypohidrosis, sparse hair, hypo-
anodontia, nail dystrophy, and frontal bossing, and




                                                 6
saddle nose deformity. In addition to abnormalities of
other ectodermally derived structures, the breast and
nipple-areolar complex may be absent or hypoplastic.




22) Which radiologic finding is associated in Fanconi's anemia?

A. posterior iliac horns

B. absent radiiCorrect Choice

C. osteopoikilosis

D. absent thumb

E. osteopathia striata
Fanconi's anemia is an inherited disease that primarily affects the bone marrow, resulting in
pancytopenia. It is also associated with a broad variety of physical anomalies. Cutaneous findings
include cafe-au-lait macules and vitiligo.


23) Birt-Hogg-Dube syndrome is most strongly associated with which of the following malignancies?

A. Renal cell carcinoma Correct Choice

B. Eccrine syringofibroadenoma

C. Trichoepithelial carcinoma

D. Medulloblastoma

E. Basal cell carcinoma
Birt-Hogg-Dube syndrome is characterized by multiple fibrofolliculomas, trichodiscomas, acro-
collagenomas, lipomas, and oral fibromas. Patients develop renal cell carcinoma, colon cancer, and
medullary thyroid carcinoma.


24) A seven month old infant diagnosed with eczema on her face returns for a diaper-rash follow-
up. A one-month trial of topical antifungals has failed to improve the infant’s systems. The part of
the physical exam that might prove most useful include:


A. Hearing test

B. Palpation of abdomen

C. Examination of palms and solesCorrect Choice

D. Stool samples




                                                  7
E. Fontanelle examination
Acrodermatitis enteropathica presents itself in infancy once breastfeeding has stopped. It can
clinically mimic atopic dermatitis, seborrheic dermatitis and candidiasis. Clinical features include
scaly red rash around mouth, eyes, and palms, diarrhea, stomatitis, glossitis, alopecia, and failure
to thrive.


25) What is the inheritance pattern of chronic granulomatous disease?

A. Autosomal dominant

B. Sporadic

C. X-linked recessiveCorrect Choice

D. X-linked dominant

E. Autosomal recessive
Chronic granulomatous disease is inherited in an x-linked recessive manner. There are mutations
present in CYBA (a cytochrome subunit), CYBB, and NCF1 & 2 (neutrophil cytosol factors 1 & 2).


26) Epidermolysis bullosa simplex is caused by blistering in which structure?

A. Spinous layer keratinocyte

B. Granular layer keratinocyte

C. Basal layer keratinocyte Correct Choice

D. Sublamina densa

E. Lamina densa
Epidermolysis bullosa simplex is caused by mutations in keratin 5 and 14 resulting in bullae within
basal cell keratinocytes.


27) Hyperextesible skin, gaping wounds, cigarette-paper scars, molluscoid pseudotumors, and
calcified subcutaneous nodules are characteristic of which syndrome?


A. Marfan syndrome

B. Congenital contractural arachnodactyly

C. Ehlers-Danlos Syndrome Correct Choice

D. Cutis laxa

E. Pseudoxanthoma elasticum
The most common type of EDS is EDS type I (gravis). Cutaneous features include hyperextensible
skin, gaping wounds, cigarette-paper scars, molluscoid pseudotumors, calcified subcutaneous
nodules, and bruises. Systemic features include hypermobile joints with dislocation, hernias, mitral
valve prolapse, blue sclerae, Gorlin’s sign (tongue reaches nose), and absence of the lingual
frenulum


28) Lamellar ichthyosis is caused by mutations in which of the following genes?

A. Phytanoyl coenzyme A hydroxylase deficiency

B. Arylsulfatase E

C. Steroid sulfatase




                                                  8
D. Transglutaminase Correct Choice

E. Fatty aldehyde oxidoreductase
Nonbullous congenital ichthyosiform erythroderma and lamellar ichthyosis are caused by mutations
in the transglutaminase gene.


29) The coast of Maine cafe au lait macule is a common finding in which of the following
syndromes?


A. Russell-Silver syndrome

B. McCune-Albright syndromeCorrect Choice

C. Watson syndrome

D. Tuberous Sclerosis

E. Neurofibromatosis type I
McCune-Albright syndrome is due to a sporadic somatic mutation in Gs subunit of adenylate cyclase.
The "coast of Maine" cafe au lait macule is a characteristic finding. Other findings in clude
polyostotic fibrous dysplasia and precocious puberty. Tuberous sclerosis, Neurofibromatosis type I,
Watson syndrome (pulmonic stenosis and CALM's) and Russell-Silver syndrome (CALMs, short
stature, precocious puberty, cryptorchidism and musculoskeletal/craniofacial defects) all have
CALMs as a feature, but not the jagged type seen in McCune-Albright syndrome.


30) Enchondromas and chondrosarcomas are most strongly associated with which of the following
syndromes?


A. Nonne-Milroy disease

B. Blue rubber bleb nevus syndrome

C. Maffucci syndrome Correct Choice

D. Osler-Weber-Rendu syndrome

E. Proteus syndrome
Maffucci syndrome is a sporadic condition caused by defects in the parathyroid hormone/parathyroid
hormone related protein type 1 receptor. There are venous malformations of distal extremities, and
benign enchondromas that can degenerate into chondrosarcomas.


31) Epistaxis in early childhood to teens followed by multiple cutaneous and gastrointestinal
telangectasias describes which of the following syndromes?


A. CREST syndrome

B. Maffucci syndrome

C. Hereditary Hemorrhagic Telangiectasia syndromeCorrect Choice

D. Ataxia telangectasia

E. Fabry disease
Hereditary Hemorrhagic Telangiectasia syndrome is described above. The first signs in over 50% of
cases is epistaxis in childhood to young adulthood. Telangectiasias develop in the 30's and 40's.
Other findings include gastrointestinal telangiectasia, hepatic and pulmonary arteriovenous
malformations. The other syndromes listed can have cutaneous vascular lesions and should be
considered on the differential for hereditary hemorrhagic telangiectasia syndrome




                                                 9
32) Which syndrome is characterized by broad thumbs, a large beaked nose, and capillary
malformation?


A. Rubinstein-TaybiCorrect Choice

B. Bloom syndrome

C. Proteus syndrome

D. Ehlers-Danlos syndrome

E. Klinefelter
Rubinstein-Taybi syndrome has been associated with a deletion localized to the short arm of
chromosome 16. Patients are severely retarded with strabismus, crytorchidism, and congenital
heart defects. They have a characteristic beaked nose with nasal septum below alae accompanied
by a broad nasal bridge, downslanting palpebral fissures, and broad thumbs and halluces.


33) Which of the following immunoglobulins is commonly decreased in Wiskott-Aldrich syndrome?

A. IgE

B. IgG

C. IgA

D. IgMCorrect Choice

E. IgD
IgM is decreased in WAS. IgA, IgD and IgE levels are all elevated. IgG is not abnormal in WAS.


34) Mutations affecting the VEGF receptor-3 cause which of the following disorders?

A. Lymphedema-distichiasis syndrome

B. Noonan syndrome

C. Lymphedema and ptosis

D. Hereditary hemorrhagic telangiectasias

E. Hereditary lymphedema (Nonne-Milroy disease) Correct Choice
Hereditary lymphedema (Nonne-Milroy disease) is an autosomal dominant condition caused by
mutations in the FLT4 gene which encodes for VEGF receptor-3. There is congenital lymphedema
and chylous ascites, scrotal swelling, intestinal tract protein loss, persistent bilateral pleural
effusion, and hypoproteinemia


35) What is a possible gene defect in this patient with a white forelock?

A. tyrosinase

B. c-kitCorrect Choice

C. P gene

D. tyrosinase-related protein 1

E. PAX3
Piebaldism is an autosomal dominant condition caused by a mutation in c-kit which is a mast and
stem cell growth factor. This mutation leads to defective melanoblast proliferation and depigmented
pathces on the trunk and white forelock. Other features include Hirschprung disease, mental
retardation, deafness and cerebellar ataxia.



                                                 10
36) Beare-Stevenson cutis gyrata syndrome is linked with mutations in:

A. ATP7A

B. BSCL2

C. None of these answers are correct

D. Fibroblast growth factor receptor 2Correct Choice

E. LMNA
Beare-Stevenson cutis gyrata syndrome has been linked to mutations in fibroblast growth factor
receptor 2. This syndrome is characterized by: craniosynostosis, ciutis gyrata, acanthosis nigricans,
anogenital anomalies, skin tags, prominent umbilical stump, furrowed palms and soles. Apert
syndrome is also linked to this mutation. BSCL2 is linked to Berardinelli-Seip congenital
lipodystrophy, LMNA to Familial partial lipodystrophy and ATP7A to Menkes kinky hair syndrome.


37) Which of the following medications is a teratogen associated with a aplasia cutis congenita?

A. Methimazole Correct Choice

B. Alcohol

C. Lithium

D. Warfarin

E. Propranolol
Aplasia cutis congenita is characterized by well-demarcated erosions at birth that heal with atrophic,
alopecic scars. Some cases are caused by medications, with methimazole considered a teratogen
particularly associated with this condition.


38) A patient with Cowden syndrome presents with a history of breast carcinoma. What other
carcinoma should she be examined/screened for?


A. Uterine

B. ThyroidCorrect Choice

C. Ovarian

D. Bone

E. Colon
Patients with Cowden syndrome are at increase risk for thyroid and breast adenocarcinomas.
Ovarian, uterine, colon and bone cancer risks are not elevated in this syndrome.


39) Which ocular finding may be seen in a patient with this skin condition?

A. comma-shaped corneal opacities

B. congenital hypertrophy of the retinal pigmented epithelium

C. retinitis pigmentosa

D. angioid streaksCorrect Choice

E. pingueculae




                                                 11
Pseudoxanthoma elasticum is caused by a defect in connective tissue. Angioid streaks develop when
a rupture occurs in Bruch;s membrane.


40) Which of the following elastic tissue diseases demonstrates calcified elastic fibers?

A. Pseudoxanthoma elasticumCorrect Choice

B. Cutis laxa

C. Buschke-ollendorf syndrome

D. Marfan syndrome

E. Anetoderma
Pseudoxanthoma elasticum is usually an autosomally recessive inherited condition due to a
defective transport protein, ABCC6. The clinical manifestations of the disease arise from fragmented
and calcified fibers of the skin, eyes and arteries. Patients may have yellow papules, loose
redundant skin, angioid streaks and hemorrhage. Histologically, the hallmark of pseudoxanthoma
elasticum is calcified elastic fibers.


41) Junctional epidermolysis bullosa with pyloric atresia is associated with mutations in:

A. Laminin 5

B. The alpha-6 subunit of integrin

C. The beta-4 subunit of integrin

D. Plectin

E. Both subunits of integrin can have mutations causing this type of junctional epidermolysis
bullosaCorrect Choice
Both subunits of integrin can have mutations causing this type of junctional epidermolysis bullosa.
Plectin is associated with epidermolysis bullosa simplex with muscular dystrophy. Laminin 5 is
mutated in Herlitz and non-Herlitz types of junctional epidermolysis bullosa.


42) Which of the following condition is NOT found in Von-Hippel Lindau syndrome?

A. Pheochromocytoma

B. Bilateral retinal hemangioblastomas

C. Renal cell carcinoma

D. Connective tissue neviCorrect Choice

E. Cerebellar/CNS hemangioblastomas
Von Hippel-Lindau syndrome is characterized by all the options listed except connective tissue nevi.
Other findings include pancreatic cysts/carcinoma and cutaneous capillary malformations of the
head and neck and polycythemia.


43) Which syndrome is due to a defective secreted mammilian Ly6/uPAR-related protein-1?

A. Netherton's syndrome

B. Mal de Meleda syndromeCorrect Choice

C. Refsum's syndrome

D. Haim-Munk syndrome




                                                  12
E. Sjogren-Larsson syndrome
Mal de Meleda, also known as keratoderma palmoplantaris transgrediens, is due to a defect in
secreted mammilian Ly6/uPAR-related protein or SLURP-1.


44) You are examining a child with mild albinism, immunodeficiency and silver grey highlights in his
hair. You diagnose the child with Chediak-Higashi syndrome. Why are you confident that this isn’t
Griscelli syndrome?


A. Griscelli syndrome has no changes in hair color

B. None of these answers are correct

C. All of these answers are correct

D. Griscelli syndrome does not have albinism as a feature

E. Giant lysosomal granules are present in neutrophils in the blood smearCorrect Choice
Chediak-Higashi syndrome and Griscelli syndrome have similar features including silver-grey
highlights of hair, immunodeficiency, mild albinism and an accelerated phase of disease. Examining
a peripheral blood smear is helpful in distinguishing between these two syndromes. Patients with
the LYST defect (a lysosomal storage transport gene) have Chediak-Higashi syndrome and will have
giant lysosomal granules visible in white blood cells on a blood smear


45) Adenosine deaminase deficiency is seen in which immunodeficient disease?

A. Wiskott-Aldrich syndrome

B. Chronic granulomatous disease

C. Leiner’s disease

D. Job syndrome

E. Severe combined immunodeficiency syndromeCorrect Choice
Severe combined immunodeficiency is a heterogeneous group of disorders characterized by
decreased humoral and cell mediated immunity. Patients may have recurrent infections including
cutaneous ones, GVHD (due to in utero cmaternal lymphocytes), sepsis, oral candidiasis, and
diarrhea. Implicated genes include the IL-2 receptor (x-linked recessive form) and adenosine
deaminase deficiency (autosomal recessive form).


46) Dystrophic epidermolysis bullosa is associated with mutations in collagen VII. Trauma or friction
induced blistering in these patients have a plane a splitting in the:


A. Stratum spinosum

B. None of these answers are correct

C. Sublamina densaCorrect Choice

D. Stratum basale

E. Lamina lucida
The split in dystrophic epidermolysis bullosa is found in the sublamina densa, where the collagen VII
anchors the epidermis to the anchoring plaques in the dermis. The remaining options are incorrect


47) What is the most likely nail findings in a patient who has this autosomal dominant disease with
these keratotic papules and cobblestoning of the oral mucosa?




                                                  13
A. Red and white longitudinal bandsCorrect Choice

B. Koilonychia

C. Pincer nails

D. Melanonychia

E. Half and half nails
Darier's disease is an autosomal dominant disorder characterized by greasy hyperkeratotic papules.
The papules often coalesce into a warty plaque and have a tendency for secondary viral or bacterial
infection. The mutation is in calcium ATPase 2A2. The classic nail finding is red and white
longitudinal bands with V-shaped nicking.


48) Low-cystine content in hair and nails may contribute to the phenotype seen in:

A. Wilson’s disease

B. Bjornstad

C. Tay SyndromeCorrect Choice

D. Menke’s Kinky Hair syndrome

E. Nethertons
Tay Syndrome is also known as trichothiodystrophy, or (P)IBIDS: (photosensitivity), icthyosis,
brittle hair, intellectual impairment, decreased fertility, and short stature. Hair shaft has a
characteristic “tiger tail” appearance under polarized light and the low cystine content in hair and
mails is thought to be responsible for the phenotype seen.


49) Bilateral ovarian cystadenomas and parotid monomorphic adenomas are seen in which of the
following conditions?


A. Beckwith-Wiederman syndrome

B. Von-Hippel-Lindau syndrome

C. Proteus syndrome Correct Choice

D. Cowden syndrome

E. Noonan syndrome
Proteus syndrome is a sporadic condition caused by postzygotic mosaic mutations in PTEN. Clinical
features include subcutaneous lymphovenous malformations, capillary malformations, lipomas,
connective tissue nevi of palms/soles, hemihypertrophy, frontal bossing, hyperostoses of epiphyses
and skull (esp. external auditory canal), scoliosis, bilateral ovarian cystadenomas, and parotid
monomorphic adenomas.


50) Maffucci syndrome is has characteristic venous malformations of the distal extremities and
benign endochondromas which can compromise bone strength and lead to chondrosarcomas. The
defect causing this believed to be the PTH/PTHrP type I receptor which is inherited in which
manner?


A. Autosomal recessive

B. X-linked recessive

C. X-linked dominant

D. Autosomal dominant




                                                  14
E. SporadicCorrect Choice
Maffucci syndrome is inherited in a sporadic manner


51) What phenotype results from a low activity of double stranded RNA adenosine deaminase?

A. oculocutaneous albinism type 4

B. Piebaldism

C. Waardenberg's syndrome type 2

D. Tietz syndrome

E. dyschromatosis symmetrica hereditariaCorrect Choice
Dyschromatosis symmetrica hereditaria (or acropigmentation symmetrica of Dohi) is an autosomal
dominant disease with hypo and hyper pigmented macules and patches on the dorsal hands and
feet associated with a low activity of double stranded RNA adenosine deaminase


52) Which of the following is not classically associated with pheochromoctyoma?

A. Multiple Endocrine Neoplasia Type IIA

B. Neurofibromatosis

C. Multiple Endocrine Neoplasia Type IIB

D. Cobb SyndromeCorrect Choice

E. Von-Hippel-Lindau Syndrome
Cobb syndrome is a sporadic disease characterized by cutaneous vascular malformations associated
with malformations of the spinal cod.


53) A 17 y/o man presents with facial acne that he would like treated. You notice that he has fine
brown scale on his neck and do a complete skin exam. This scale is present on the remainder of his
body, sparing his palms, soles and flexural areas. He informs you that his uncles on his mother’s
side have similar skin findings. He is not concerned about the skin and would like to proceed with
acne treatment only. What other clinical exam should you perform to screen for a potential
malignancy to which this patient is at higher risk for acquiring?


A. Lung

B. Rectal

C. Abdominal

D. TesticularCorrect Choice

E. Lymph node
Men with x-linked ichthyosis are at increased risk of testicular cancer and cryptorchidism. A
testicular exam is simple to perform and a good screening exam for detecting testicular
abnormalities. The remaining exams are not useful as these patients are not at higher risk for other
types of cancer


54) Naxos syndrome is characterized by a right sided cardiomyopathy, wooly hair, and
keratoderma. The epidermal structure defective in Naxos syndrome is:


A. desmoplakin

B. plakoglobinCorrect Choice



                                                 15
C. desmoglein 3

D. desmoglein 1

E. Keratin 1/10
Plakoglobin is an intracellular desmosomal component which binds desmogleins/desmocollins on one
side and to desmoplakin on the other. Desmoplakin in turn binds to the keratin intermediate
filaments, K1/10 in most cases. Mutation of desmoplakin leads to CarvajaL syndrome, which is
associated with a striate palmoplantar keratoderma, woolly hair and Left sided cardiomyopathy. A
simple way to remember this is the L in Carvajal cooresponds to the Left sided cardiomyopathy vs.
the right sided disease in Naxos disease.


55) What cutaneous manifestation is associated with familial cerebral cavernomas?

A. hyperkeratotic cutaneous capillary-venous malformations(HCCVM)Correct Choice

B. Verrucous hemangioms

C. segmental facial hemangiomas

D. Tufted angiomas

E. Glomeruloid hemangiomas
Familial cerebral cavernomas are due to a defect in the CCM gene which encodes the KRIT-1
protein. These patients often times have hyperkeratotic cutaneous capillary-venous malformations.


56) "Mousy" odor of urine is characteristic of which of the following syndromes?

A. Maple Syrup Urine disease

B. Hunter disease

C. Alkaptonuria

D. Hurler disease

E. PhenylketonuriaCorrect Choice
Phenylketonuria will cause a "mousy" odor in the urine. Patients with Alkaptonuria will have black
urine. Maple syrup urine disease will have a sweet odor. Hunter/Hurler diseases are not associated
with urinary issues.


57) Which disease can clinically mimic pellagra but is inherited in an autosomal recessive fashion
and is due to a defect in the transport of neutral amino acids?


A. Fabry

B. Wilsons

C. Gaucher’s

D. Hartnup DiseaseCorrect Choice

E. Hemochromatosis
The clinical manifestation of Hartnup disease is similar to that of pellagra because the resultant
defect in the transport of amino acids leads to low levels of tryptophan. Since tryptophan is required
to make nicotinic acid, pts with Hartnup disease manifest the same symptoms as niacin-deficient
patients (pellagra).




                                                 16
58) A child presents with the hair finding seen in the image in addition to brittle nails, keratosis
pilaris, abnormal teeth and cataracts. Which of the following abnormalities is the most likely
mutated?


A. Keratin hHb1/hHb6Correct Choice

B. Keratin 1/10

C. Keratin 2e

D. Keratin 6/17

E. Keratin 6/16
Keratin hHb1/hHb6 is defective in monilethrix, which is described above. Keratin 1/10 defects are
found in epidermolytic hyperkeratosis, K6/16 in inflamed skin and pachyonychia congenita type I,
K6/17 in pachyonychia congenita type II and K2e in Ichythosis bullosa of Siemens


59) Which malignancy is associated with Cowden syndrome?

A. Colon cancerCorrect Choice

B. Renal cancer

C. Lung cancer

D. Basal cell cancer

E. Ovarian cancer
Cowden syndrome is a autosomal dominant syndome with tricholemmomas, oral mucosal papules,
acral keratotic papules, thyroid goiter, GI polyps, and fibrocystic breast disease. Malignant
associations include breast, thyroid follicular carcinoma, and colon adenocarcinoma.


60) The combination of gastrointestinal polyposis, nail atrophy, alopecia, generalized pigmentation
of skin, and melanotic macules of the fingers is characteristic of which of the following syndromes?


A. Cronkhite-Canada syndrome Correct Choice

B. Bannayan-Riley-Ruvalcaba syndrome

C. Cowden syndrome

D. Nicolau-Balus syndrome

E. Peutz-Jeghers syndrome
Cronkhite-Canada syndrome is a sporadic gastrointestinal polyposis syndrome associated with nail
atrophy, alopecia, generalized pigmentation of the skin, and melanotic macules on the fingers.


61) Which of the following syndromes is associated with cutis marmorata?

A. Netherton’s

B. Cornelia de Lange syndromeCorrect Choice

C. Hemansky-Pudlak syndrome

D. Maffucci syndrome

E. Papillon-Lefevre syndrome
Cornelia de Lange is also known as Brachmann-de Lange syndrome. Cutaneous manifestations
include cutis marmorata, hirsutism, hypoplastic nipples and umbilicus. Patients also have small




                                                   17
hands and feet. They have characteristic facies which include hirsutism on the forehead,
trichomegaly, synophrys, anteverted nostrils, long philtrum, and low-set ears.


62) Lamellar ichthyosis is caused by a defect in transglutaminase 1. It can present as a colloidion
baby at birth with subsequent large thick plates of scale especially on flexures, ectropion and
eclabium. If two unaffected carrier parents have a child, how likely is their child to have this
condition?


A. Only male offspring are affected

B. None of these answers are correct

C. Only female offspring are affected

D. 25%Correct Choice

E. 75%
Lamellar ichthyosis is an autosomal recessive condition. If each parent is a heterozygous carrier,
there is a 25% chance that the child will be affected, 50% chance that the child will be a
heterozygous carrier and a 25% chance that the child will not be a carrier or affected with lamellar
ichthyosis.


63) A 18 yo man presents for evaluation of foot lesions. There are thick hyperkeratotic plaques
symmetrically on only the weight bearing plantar surfaces. What test(s) should this patient be
referred for?


A. Thoracic CT

B. Head CT

C. EndoscopyCorrect Choice

D. Hepatic ultrasound

E. Knee films
The patient likley has Howel-Evans Syndrome. These patients present with symmetric focal weight
bearing PPK in the second decade to adulthood. After the third decade, esophageal carcinoma can
occur. These patients should have periodic endoscopic evaluation.


64) To help diagnose trichothiodystrophy, which of the following levels are decreased in hairs of
affected individuals?


A. Histidine

B. Arginine

C. CysteineCorrect Choice

D. Glycine

E. Phenylalanine
Cysteine and Methionine levels are decreased in hair and nails of patients with trichothiodystrophy.
Other sulfur containing amino acids include: Gluthathione, Taurine, and Homocysteine. Testing hairs
for decreased sulfur content is an indirect method of determining this. The other listed amino acids
are present in normal levels in the hair and nails of trichothiodystrophy patients


65) Adenosine deaminase deficiency is associated with which of the following disorders?

A. Xeroderma pigmentosum




                                                  18
B. Wiskott Aldrich syndrome

C. Gout

D. Severe combined immunodeficiencyCorrect Choice

E. Job syndrome
Adenosine deaminase deficiency is associated with severe combined immunodeficiency. The most
common inheritance is x-linked recessive. It is a mixed group of disorders all sharing defects in cell-
mediated and humoral immunity. Skin findings include: candidal infections, mucocutaneous,
bacterial pyodermas, seborrheic-like dermatitis/lichen planus-like sclerodermatous changes, aplastic
thymus and pneumonias. The other listed syndromes are not associated with adenosine deaminase
deficiency.


66) Which eye findings would be expected in an individual with this disorder associated with
atherosclerosis?


A. dendritic corneal ulcerations

B. salt and pepper retinitis pigmentosa

C. angoid streaksCorrect Choice

D. keratoconus

E. ectopia lentis
Pseudoxanthoma elasticum is an autosomal rescessive or autosomal domminant disease caused by
a mutation in ABCC6 (adenosine triphosphate-binding cassette subfamily C member 6). Associated
findings include gastric artery hemorrhage, angiod streaks, retinal hemorrhage, atherosclerotic
disease, and a possible increased risk of first trimester miscarriage.


67) Which of the following is caused by a mutation in a gene which codes for steroid sulfatase?

A. Lamellar icthyosis

B. Chediak-Higashi

C. X-linked ichthyosisCorrect Choice

D. Lesch-nyhan disease

E. Fabry disease
The mutation in X-linked icthyosis is found in the gene for aryl sulfatase C, a steroid sulfatase. The
genetic defect in Fabry disease occurs in alpha-galactosidase A (which hydrolyzes glycolipids and
glycoproteins), the defect in lamellar icthyosis codes for transglutaminase 1 and the defect in
chediak higashi occurs in a lysosomal transport protein.


68) Meleda is an island off the coast of Croatia. Its inhabitants have an increased frequency of a
malodorous transgradiens palmoplantar keratoderma in a stocking-glove distribution. This condition
is autosomal recessive with a defect in:


A. Cathepsin C

B. Keratin 1/9

C. SLURP1 geneCorrect Choice

D. TOC gene

E. Loricrin




                                                  19
SLURP1 (Secreted LY6/UPAR-related protein 1) is defective in Mal de Meleda. Keratin 1/9 are
defective in Unna-Vorner/Thost palmoplantar keratoderma, an AD diffuse symmetric non-
transgradiens PPK. TOC gene is defective in Howel-Evans syndrome, an AD PPK associated with
focal, pressure-related, non-transgradiens PPK and esophageal cancer/oral leukoplakia. Loricrin
mutations are seen in Vohwinkel syndrome variant and symmetric progressive erythrokeratodermia.
Cathepsin C defects are seen in Haim-Munk syndrome
(PPK+periodontitis+acroosteolysis+onychogryphosis) and Papillon-Lefevre syndrome (sharply
demarcated transgradiens, stocking-glove PPK+periodontitis+dural calcifications and choroids
attachments)


69) A patient with thyroid carcinoma and cobblestone-like changes of the oral mucosa will also
likely have:


A. Syringomas

B. Cylindromas

C. Trichoepitheliomas

D. Fibrofolliculomas

E. TricholemmomasCorrect Choice
The patient described may have Cowden's syndrome, an autosomal dominant condition caused by a
defect in the PTEN tumor suppressor gene. Patients with Cowden's disease are at increased risk for
thyroid and breast carcinoma. In addition, they characteristically have multiple hamartomatous
polyps of the gastrointestinal tract that are typically benign. Cutaneous features of Cowden's
syndrome which may serve as clues to the diagnosis include multiple oral papillomas with a
"cobblestone" appearance on the lips, gingival, and buccal mucosa, acral keratotic papules on the
dorsal hands and wrists, palmoplantar punctate keratoses and multiple facial tricholemmomas.
Patients with this condition need careful malignancy surveillance.


70) What is the inheritance pattern of dermatosis with acantholysis?

A. autosomal recessive

B. X-linked recessive

C. X-linked dominant

D. autosomal dominantCorrect Choice

E. sporadic
Hailey-Hailey, or Familial Benign Pemphigus, is an autosomal dominant genodermatosis which is
caused by a mutation in ATP2C1. Vesicles and erythematous plaques develop in the skin folds such
as axillae and groin area.


71) A patient with myotonic dystrophy and multiple skin lesions characterized by the pathology
image shown most likely has activating mutations in which of the following?


A. desmoglein

B. desmoplakin

C. plakoglobin

D. beta-cateninCorrect Choice

E. alpha 6-beta 4 integrin
Myotonic dystrophy with multiple pilomatricomas is described above. Activating mutations in Beta-
catenin are found in this syndrome. The other listed options are desmosomal proteins and are not
involved in this syndrome.



                                                20
72) Painful crises and 'whorled' corneal opacities are seen with which of the following enzyme
abnormalities?


A. alpha-galactosidase ACorrect Choice

B. glucocerebrosidase

C. iduronate sulfatase

D. glucoronidase

E. homogentisic acid oxidase
Painful crises and whorled corneal opacities are found in Fabry disease which is caused by a defect
in alpha-galactosidase A. The remaining conditions do not have these findings


73) A patient with melanoma and a malignant glioma is diagnosed with Li-Fraumeni syndrome.
Which of the following tumors occurs most frequently in this disease?


A. Breast carcinoma

B. Lung carcinoma

C. RhabdomyosarcomaCorrect Choice

D. Leukemia

E. Adrenocortical carcinoma
Li-Fraumeni syndrome is a familial tumor syndrome caused by mutations in the tumor suppressor
gene p53. They are at risk for a wide range of malignancies with particularly high occurrences of
soft tissue sarcomas, breast caner, brain tumors, acute leukemia, and adrenal cortical carcinoma.
Soft tissue sarcomas are among the most common reported with this disease.


74) Most common malignancy to develop in a patient with tricholemmomas, acral verrucous
papules and cobble-stoning of buccal and gingival mucosa?


A. Colon cancer

B. Breast cancer

C. Melanoma

D. Lymphoma

E. Thyroid cancerCorrect Choice
Cowden's syndrome is an autosomally dominant inherited defect of PTEN. Patient may present with
multiple trichilemmoma, hamartomatous tumors of the breast, thyroid and endometrium, acral
keratoses and papillomatous papules. Thyroid carcinoma is the most common form malignancy to
arise in these patients


75) You are consulted on a patient with possible Nethertons Syndrome. Which location of the body
would most likely have hairs demonstrating trichorrhexis invaginata?


A. none of these answers are correct

B. scalp

C. eyebrowCorrect Choice

D. all of these answers are correct




                                                 21
E. eyelash
Eyebrow hair is most common site with hairs demonstrating trichorrhexis invaginata.


76) Beckwith-Wiederman syndrome is characterized by which of the following triads?

A. Epistaxis, telangictases, and gastrointestinal tract bleeding

B. Omphalocele, venous malformations, and ataxia

C. Exophthalmos, macroglossia, and gigantism Correct Choice

D. Hemangioblastomas, renal cysts and renal cell carcinoma

E. Enlarged limb, port wine stain, and deep venous thrombosis
Beckwith-Wiederman syndrome is also known as EMG syndrome as it includes exophthalmos,
macroglossia, and gigantism. It is usually a sporadic condition but is sometimes caused by
autosomal dominant mutations in p57. Clinical features include facial capillary malformations,
macroglossia, visceromegaly with omphalocele, and hemihypertrophy associated with tumors
(especially Wilm’s tumors).


77) A 16-month old girl presents with patchy alopecia, whorled erythematous scaly eruption, and
asymmetric limb shortening. What laboratory or radiologic test may aid in diagnosis?


A. Alkaline phosphatase

B. Brain MRI

C. Bone filmsCorrect Choice

D. Chest radiograph

E. Complete blood count
The patient has Conradi-Hunermann Syndrome. This is a X-linked dominant disorder characterized
by ichthyosiform erythroderma in Blaschko's lines in infancy which resolves with follicular
atrophoderma, patchy alopecia, short stature, cataracts, scoliosis, assymetric limb shortening. Bone
films will demonstrate stippled epiphyses. Ichthyosis and stippled epiphyses resolve after infancy.


78) A patient presents with multiple flesh colored papules on his face. The pathology report comes
back as a fibrofolliculoma. He also has multiple lipomas. Which of the following neoplasms must you
be concerned about?


A. Renal Cell Carcinoma

B. Colon carcinoma

C. All of the options are correctCorrect Choice

D. Medullary thyroid carcinoma

E. None of the options are correct
Birt-Hogg-Dube syndrome is described above. Renal cell carcinoma is the most common association
for internal malignancy, but medullary thyroid and colon carcinomas occur at an increased rate in
these individuals also


79) Menke’s kinky hair syndrome is caused by a defect in:

A. Gap junction protein

B. Mitochondrial gene



                                                   22
C. Copper Transporting ATPaseCorrect Choice

D. DNA helicase

E. Proto-oncogene
Menke’s kinky hair syndrome is an x-linked recessive disorder caused by a mutation at Xq12 leading
to defective intestinal copper transport


80) Retinal hemangioblastomas are found in which syndrome:

A. Kasabach-Merritt syndrome

B. Klippel-Trenaunay Weber syndrome

C. Sturge-Weber syndrome

D. Von-Hippel Lindau diseaseCorrect Choice

E. Osler-Weber-Rendu disease
Von Hippel-Lindau syndrome is an autosomal dominant condition caused by a defect in the VHL
tumor suppressor gene. This disease is characterized by retinal hemangioblastomas, often resulting
in visual impairment and blindness if left untreated. In addition, many tumors are seen including
pheochromocytoma, renal cell carcinoma, and hemangioblastomas of the cerebellum, medulla, and
spinal cord. Pancreatic and renal cysts are also a feature of this condition. Finally, polycythemia can
occur as a result of erythropoietin production by renal cell carcinoma. Von Hippel-Lindau syndrome
is a progressive, universally fatal condition which presents most often in the fourth decade of life


81) Mutations in calcium transporters cause which pair of diseases?

A. Chondrodysplasia punctata and CHILD syndrome

B. Darier’s disease and Hailey-Hailey disease Correct Choice

C. Lamellar ichthyosis and nonbullous congenital ichthyosiform erythroderma

D. Refsum syndrome and Sjogren-Larsson syndrome

E. Erythrokeratodermia variabilis and progressive symmetric erythrokeratodermia
Darier’s disease and Hailey-Hailey disease are caused by mutations in the SERCA calcium ATPase.
The former is characterized by hyperkeratotic papules in seborrheic areas, palmar keratoses and
pits, red-white longitudinal nail bands, v-shaped distal nail nicks, and cobblestoning of oral and
rectal mucosae. The latter is characterized by acantholytic erosions in skin folds


82) Which of the following is a potentially serious complication of the blue rubber bleb nevus
syndrome?


A. Development of lymphedema

B. Gastrointestinal hemorrhage Correct Choice

C. Development of chondrosarcomas

D. Development of lymphedema

E. Development of angiosarcomas
Blue rubber bleb nevus syndrome is characterized by multiple tender venous malformations of skin
and gastrointestinal tract, which can lead to gastrointestinal bleeding


83) A child with phenylketonuria likely presents with which cutaneous problems?



                                                  23
A. Alopecia universalis

B. Generalized hypopigmentationCorrect Choice

C. Leg ulcers

D. Blue-gray generalized hyperpigmentation

E. Generalized hyperpigmentation
Phenylketonuria is an autsomal recessive disorder caused by a mutation on the long arm of
chromosome 12. A deficiency of phenylalanine hydroxylase or its cofactor tetrahydrobiopterin leads
to accumulation of phenylalanine. Clinical features include generalized hypopigmentation,
eczematous dermatitis, sclerodermoid changes, seizures, psychomotor delay, urine with “mousy”
odor, mental retardation


84) Patients with Chondrodysplasia punctata can have findings of stippled epiphyses on X-ray
examination. Which other x-linked dominant condition can have stippled epiphyses?


A. CHILD syndromeCorrect Choice

B. Bazex syndrome

C. Incontinentia Pigmenti

D. Goltz syndrome

E. Focal Dermal Hypoplasia
All of the syndromes listed have X-linked dominant inheritance. CHILD syndome also has findings of
stippled epiphyses. Incontinentia pigmenti is caused by defecdts in the NEMO gene. Findings include
peg/conical teeth, eye and CNS defects and alopecia. There are no bone abnormalities. Focal
Dermal Hypoplasia, otherwise known as Goltz syndrome has findings of linear atrophy following
Blaschko's lines with areas of fat herniation, mucocutaneous papillomas and pits, alopecia, nail
dystrophy, tooth abnormalities and osteopathia striata (striations of the long bones). Bazex
syndrome is associated with follicular atrophoderma, hypohidrosis, hypotrichosis and multiple basal
cell carcinomas. There are no bone abnormalities associated


85) Comma-shaped corneal opacities are characteristic of which type of ichthyosis?

A. X-linked ichthyosis Correct Choice

B. Nonbullous congenital ichthyosiform erythroderma

C. Lamellar ichthyosis

D. Ichthyosis vulgaris

E. Refsum syndrome
X-linked ichthyosis patients have comma-shaped corneal opacities that are asymptomatic yet highly
characteristic.


86) Painful crises and 'whorled' corneal opacities are seen with which of the following enzyme
abnormalities?


A. iduronate sulfatase

B. glucoronidase

C. glucocerebrosidase

D. alpha-galactosidase ACorrect Choice




                                                24
E. homogentisic acid oxidase
Painful crises and whorled corneal opacities are found in Fabry disease which is caused by a defect
in alpha-galactosidase A. The remaining conditions do not have these findings.


87) In chronic granulomatous disease, the diagnosis is made by which of the following tests?

A. Assay for fumarate hydratase

B. Nitroblue tetrazolium reduction assayCorrect Choice

C. Assay for sphingomyelinase

D. Skin biopsy

E. Assay for glucocerebrosidase
Chronic granulomatous disease is diagnosed by the nitroblue tetrazolium reduction assay. The
abnormal white blood cells cannot reduce dye due to the inability to produce the respiratory burst.
This is needed to kill catalase positive organisms after phagocytosis. Fumarate hydratase is
defective in familial multiple cutaneous leiomyomatosis, sphingomyelinase in Niemann-Pick disease
and glucocerebrosidase in Gaucher disease


88) Milia, atrophoderma vermiculata and eruptive lesions shown in the pathologic image are
characteristic of which of the following syndromes?


A. Banayan-Riley-Ruvalcaba

B. Gorlin

C. Nicolau-BalusCorrect Choice

D. Cowden

E. Birt-Hogg-Dube
Nicolau-Balus syndrome is characterized by eruptive syringomas (shown in the path image), milia
an datrophoderma vermiculata. Cowden syndrome is associated with trichilemmomas, Birt-Hogg-
Dube with fibrofolliculoma, and Banayan-Riley-Ruvalcaba is not associated with an adnexal
neoplasm


89) Which of the following bony defect is found in CHILD syndrome?

A. Calcification of falx cerebri

B. Osteopoikilosis

C. Stippled epiphysesCorrect Choice

D. Sphenoid wing dysplasia

E. Polyostotic fibrous dysplasia
CHILD syndrome is an X-linked dominant syndrome which is lethal in males. It is caused by a
peroxisomal biogenesis disorder. It is characterized by unilateral ichthyosiform erythroderma, limb/
visceral hypoplasia, and stippled epiphyses. Stippled epiphyses can also be seen in
chondrodysplasia punctata. Polyostotic fibrous dysplasia is found in McCune-Albright syndrome,
calcification of falx cerebri in Gorlin's syndrome, osteopoikilosis in seen in Buschke-Ollendorf
syndrome


90) Epidermolysis bullosa with muscular dystrophy is caused by mutations in which of the
following?




                                                 25
A. Keratins 5 and 14

B. Loricrin

C. Collagen 17

D. Plectin Correct Choice

E. Collagen 7
Epidermolysis bullosa with muscular dystrophy is caused by mutations in plectin


91)
Which of the following syndromes is associated with
hematologic abnormalities?


 A. Klippel-Trenaunay-Parks-Weber

 B. Kasabach-Merritt syndromeCorrect Choice

 C. Bloom’s syndrome

 D. Blue rubber bleb nevus syndrome

 E. Sturge Weber disease
Kasabach-Merritt syndrome is associated with
hematologic abnormalities, such as
thrombocytopenia, microangiopathic hemolytic
anemia, disseminated intravascular coagulation. The
condition develops from platelet-trapping within a
large hemangioma, most commonly a kaposiform
hemangioendothelioma in the retroperitoneal
location.


92) Which of the following is caused by a defect in lysosomal transport:

A. Chediak-higashiCorrect Choice

B. oculocutaneous albinism II

C. Piebaldism

D. oculocutaneous albinism I

E. Bloom’s syndrome
Chediak higashi is an autosomal recessive disorder caused by a mutation in the LYST gene codes for
a lysosomal tracking protein. This protein regulates microtubule mediated lysosomal fusion. A defect
in this gene leads to giant lysosomal granules seen in neutrophils (leading to defecting phagocytosis
and decreased chemotaxis), melanocytes (pigment dilution), and neurons. OCA1 is tyrosinase
negative albinism; OCA2 is tyrosinase positive albinism with a mutation in P gene on chromosome
15


93) Which keratins are expressed in the suprabasal palmoplantar epidermis?

A. Keratins 4 and 13

B. Keratins 8 and 18

C. Keratins 1 and 9 Correct Choice

D. Keratins 5 and 14




                                                 26
E. Keratins 1 and 10
Keratins are expressed in pairs of acidic and basic keratins and are tissue- and differentiation-
specific. Keratins 1 and 9 are expressed in the suprabasal palmoplantar epidermis, keratins 1 and
10 in suprabasal nonpalmoplantar epidermis, keratins 2e and 10 in the granular layer, keratins 4
and 13 in mucosal epithelium, keratins 5 and 14 in the basal layer, and keratins 8 and 18 in simple
epithelium.


94) Dermatofibrosis lenticularis disseminata is seen in which of the following conditions?

A. Pseudoxanthoma elasticum

B. Focal dermal hypoplasia

C. Ehlers-Danlos syndrome

D. Marfan syndrome

E. Buschke-Ollendorf syndrome Correct Choice
Buschke-Ollendorf syndrome is an autosomal dominant disorder characterized by dermatofibrosis
lenticularis disseminata (cutaneous elastomas distributed symmetrically over the buttocks, trunk
and proximal extremities), and osteopoikilosis (round opacities in bones


95) A patient with 20 nail dystrophy, steatocystoma multiplex and natal teeth likely has a mutation
in the genes coding for:


A. Laminin 5

B. Plakophilin 1

C. Keratins 5 &14

D. Keratins 6b & 17Correct Choice

E. Keratins 6 &16
Pachyonychia congenital is an autosomal dominant condition with 20 nail dystrophy. The patient
described has Type II (Jackson-Sertole) disease, which includes steatocystoma multiplex, natal
teeth, multiple cysts, and micropthalmia, and is caused by mutations in keratins 6b& 17. Type I
(Jadassohn-Lewandowsky) also includes focal symmetric PPK, follicular hyperkeratosis, oral
leukokeratoses and is caused by mutations in keratins 6 &16. Type III includes the clinical features
of type I + corneal leukokeratosis. Mutations in keratins 5&14 represents EB simplex, Laminin 5
mutation is seen in Junctional EB, and plakophilin 1 mutation is seen in ectodermal dysplasia with
skin fragility


96) Premalignant leukoplakia of the oral mucosa is associated with:

A. Werner Syndrome

B. Bloom syndrome

C. Rothmund-Thomson syndrome

D. Dyskeratosis CongenitaCorrect Choice

E. Xeroderma Pigmentosum
Dyskeratosis Congenita (also known as Zinsser-Engman-Cole syndrome) is thought to have two
modes of inheritance. The more common X-linked disorder is due to a mutation in the Dyskerin
gene, while the autosomal dominant form is due to a mutation in TERC, a telomerase RNA
component. Clinical features include reticulated gray-brown hyperpigmentation, paloplantar
hyperkeratosis, alopecia, onychodystrophy, premalignant leukoplakia of any mucosal surface, and
mental retardation



                                                 27
97) Which of the following disorders is associated with delayed separation of the umbilical cord?

A. Leukocyte adhesion deficiency type 1 (LAD-1)Correct Choice

B. Myeloperoxidase deficiency

C. X-linked agammaglobulinemia

D. Severe combined immunodeficiency disorder(SCID)

E. Immunedysregulation, polyendocrinopathy, enteropathy, x-linked (IPEX)
LAD-1 manifests as a B2-integrin deficiency and often times presents as at birth with a delayed
umbilical cord separation.


98) Mucosal neuromas, pheochromocytoma and medullary thyroid carcinoma in a patient with a
marfanoid body habitus is associated with which of the following gene defects?


A. RET proto-oncogeneCorrect Choice

B. STK11

C. BHD

D. Menin

E. PTEN
The RET proto-oncogene is mutated in Multiple Endocrine Neoplasia type II. Type IIb is described
above. Other findings include rare parathyroid abnormalities, megacolon, thickened lips and thick,
everted upper eyelids. Menin is associated with MEN type I, PTEN with Cowden disease, BHD with
Birt-Hogg-Dube syndrome and STK11 with Peutz-Jeghers syndrome


99) In a patient suspected of having multiple endocrine neoplasia type IIb, which lab test would be
appropriate?


A. Parathyroid hormone

B. Cortisol

C. CalcitoninCorrect Choice

D. Glucagon

E. Calcium
Multiple endocrine neoplasia (MEN) syndrome type Iib, also called multiple mucosal neuroma
syndrome is an autosomal dominant condition due to a defect in the RET protooncogene on
chromosome 10q11.2. This rare condition is associated with mucosal neuromas on the tongue and
lips, medullary thyroid carcinoma, pheochromocytoma, and gastrointestinal ganglioneuromatosis. In
addition, patients can also present with a marfanoid habitus and facial dysmorphism. Mucosal
neuromas can be a dermatologic clue to the underlying diagnosis as these lesions appear during
early childhood and present as pink, pedunculated nodules. As the major cause of mortality in these
patients is medullary thyroid cancer, which nearly all patients will have by early adulthood,
aggressive screening, with serial calcitonin level, and prophylactic thyroidectomy are warranted.


100) What is the gene defect in this condition, which is also called Mendes da Costa syndrome?

A. keratin 1 and 10

B. SPINK5




                                                 28
C. connexin 31Correct Choice

D. SLURP-1

E. connexin 26
Mendes da COsta syndrome is also called Erythrokeratoderma Variabilis. It is an autosomal
dominantly inherited due to a mutation in connexin 31 or connexin 30.3. It is characterized by
transient geographic patches of erythema and hyperkeratotic plaques


101) The hair abnormality shown in the image is characteristic of which of the following diseases?

A. TrichothiodystrophyCorrect Choice

B. Monilethrix

C. Bjornstad syndrome

D. All of the options are correct

E. Naxos disease
The hair abnormality shown is trichoschisis, clean breaks of the hair shaft which occurs in
trichothiodystrophy. Also commonly seen is the "tiger-tail" banding pattern of the hair when placed
under polarized light. The other options do have hair shaft abnormalities, but not trichoschisis


102) The treatment for acrodermatitis enteropathica is:

A. Vitamin B12 supplementation

B. Zinc supplementationCorrect Choice

C. Phlebotomy

D. Iron supplementation

E. Vitamin B1 supplementation
Acrodermatitis enteropathica is due to a defect in zinc absorption and will respond to zinc
supplementation. Iron, Vitamin B1/12 supplementation will not result in improvement in this
condition. Findings include periorificial, scalp, and acral dermatitis, scaling, vesicles/bullae, erosions,
alopecia, diarrhea and stomatitis


103) In patients with diffuse congenital hemangiomatosis, the most common site for
extracutaneous involvement is the :


A. Brain

B. Thyroid

C. LiverCorrect Choice

D. Lungs

E. Colon
Diffuse congenital hemangiomatosis is characterized by multiple hemangiomas with the liver being
the most common extracutaneous site, followed by the lungs. Liver hemangioma may be
complicated by hepatomegaly, obstructive jaundice, and portal hypertension


104) A deficiency in sialophorin, a surface glycoprotein, is thought to play a role in which genetic
disorder?




                                                    29
A. Bloom Syndrome

B. Chediak-higashi syndrome

C. Cornelia de Lange syndrome

D. Werner syndrome

E. Wiskott-aldrich syndromeCorrect Choice
Wiskott-Aldrich syndrome is an x-liked recessive disorder localized to Xp11.3. The gene involved
codes for WAS, and the protein product has been implicated in lymphocyte and megakaryocyte
signal transduction


105) Sphenoid wing dysplasia is seen in:

A. Tay Syndrome

B. NF-1Correct Choice

C. NF-2

D. Mafucci syndrome

E. Tuberous sclerosis
Sphenoid wing dysplasia is seen in neurofibromatosis type I. Patients with Tay syndrome have short
stature, patients with tuberous sclerosis have phalangeal cysts and periosteal thickening, patients
with Mafucci syndrome have enchondromas and short stature, and patients with NF-2 do not have
any characteristic musculoskeletal findings


106) A patient Buschke-Ollendorff syndrome has osteopoikilosis and which cutaneous finding?

A. Epidermal nevi

B. Café au lait macules

C. Waxy papules along the eyelids

D. Port wine stain

E. Juvenile elastomaCorrect Choice
Buschke-Ollendorf syndrome is an autosomal dominant syndrome associated with increased elastic
fiber in the skin. Key features include dermatofibrosis lenticularis disseminata (also called juvenile
elastomas) and osteopoikilosis


107) Which of the following is NOT part of the Carney complex?

A. Endocrine abnormalities

B. Peg or conical teethCorrect Choice

C. Pigmented skin lesions

D. Cardiac, cutaneous or mammary myxomas

E. Primary pigmented nodular adrenocortical disease
Peg/conical teeth are not part of the Carney complex. This is found in incontinentia pigmenti and
anhidrotic ectodermal dysplasia. The remaining skin findings are part of this complex sometimes
known as NAME syndrome. It consists of multiple, diffuse mucocutaneous lentigines, cardiac and
subcutaneous myxomas and endocrine abnormalities may be present. Other findings include:
testicular tumors, thyroid disease, primary pigmented nodular adrenocortical disease,
psammomatous melanotic schwannomas and hormone-secreting pituitary adenomas




                                                  30
108) A patient presents with the multiple painful papules on the abdomen with pathology as shown
in the image. They report that their father had similar skin lesions. What enzyme defect is the most
likely cause?


A. Steroid sulfatase

B. homogentisic acid oxidase

C. glucocerebrosidase

D. alpha-galactosidase

E. fumarate hydrataseCorrect Choice
Fumarate hydratase has been implicated in familial multiple cutaneous leiomyomatosis. Steroid
sulfatase is defective in x-linked ichthyosis, alpha-galactosidase in Fabry disease, homogentisic acid
oxidase in alkaptonuria and glucocerebrosidase in Gaucher disease.


109) Cutaneous osteomas are seen in which syndrome?

A. Carney complex

B. Albright hereditary osteodystrophy Correct Choice

C. Gaucher’s syndrome

D. Waardenburg syndrome

E. LEOPARD syndrome
Albright hereditary osteodystrophy is caused by mutations in the Gs subunit of adenylate cyclase.
There is calcification and ossification due to pseudohypoparathyroidism, absent 4th knuckle, and
hypogonadism.


110) What is the genetic defect of this autosomal dominant disorder?

A. Calcium ATPase 2C1Correct Choice

B. PTPN11

C. SPINK5

D. Calcium ATPase 2A2

E. PTEN
Hailey-Hailey is an autosomal dominant disorder that usually affects the intertriginous areas.
Clinically, there is erythema and linear fissures of the axilla and groin. On pathology, the
characteristic finding is dyskeratosis in a "dilapidated brick wall" pattern. The gene defect
responsible is calcium ATPase 2C1


111) Primary pigmented nodular adrenocortical disease and psammomatous melanotic
schwannomas are characteristic of which of the following syndromes?


A. McCune-Albright syndrome

B. Tuberous sclerosis

C. Gaucher’s syndrome

D. Hypomelanosis of Ito




                                                 31
E. Carney complex Correct Choice
Carney complex is an autosomal dominant disorder caused by mutations in PRKAR1A (protein
kinase A regulatory subunit 1-alpha). Key features include cardiac, cutaneous and mammary
myxomas, pigmented skin lesions, endocrine abnormalities (pituitary, testicular, thyroid, etc),
primary pigmented nodular adrenocortical disease, and psammomatous melanotic schwannomas


112) A triangular-shaped lunula is a characteristic finding in which disease?

A. incontinentia pigmenti

B. Darier's disease

C. dyskeratosis congenita

D. nail-patella syndromeCorrect Choice

E. epidermal nevus syndrome
Nail-patella syndrome, also called hereditary osteo-onychodysplasia is a rare autosomal dominant
condition caused by a defect in the LMX1B gene. It is characterized by triangular lunulae,
palmoplantar hyperhidrosis, renal dysplasia, glomerulonephritis, and hyperpigmentation of the
papillary margin of the iris, an ophthalmologic finding also known as Lester iris. Other nail findings
include micronychia with hemionychia, anonychia, and longitudinal fissures. Bony findings include
absent or hypoplastic patella, posterior iliac horns, radial head subluxation, thickened scapulae, and
scoliosis.
Nail findings in Darier's disease include red and white longitudinal bands, subungual hyperkeratosis
and V-shaped nicking of the distal nail plate. Dystrophic nails with longitudinal ridges, pterygium,
and atrophic or absent nails can be found in dyskeratosis congenita. Dystrophic changes of the nails
can be seen in approximately 5-10% of patients with incontinentia pigmenti.


113) A patient with renal cell carcinoma caused by mutations in fumarate hydratase deficiency
likely suffers which of the following conditions?


A. Multiple endocrine neoplasia

B. Birt-Hogg-Dube syndrome

C. Von-Hippel-Lindau syndrome

D. Familial multiple cutaneous leiomyomatosis Correct Choice

E. Cowden syndrome
Familial multiple cutaneous leiomyomatosis is an autosomal dominant condition caused by
mutations in the fumarate hydratase gene. Clinically, there are multiple cutaneous leiomyomas,
uterine leiomyomas and leiomyosarcomas, as well as renal cell carcinomas


114) A patient presents with several light blue cyst-like lesions on the eyelid. They consult their list
of problems and bring up plantar hyperkeratosis and dysplastic toenails. On oral exam, you note
that they have both upper and lower dentures. The patient relates that after losing their "baby
teeth", only 3 teeth grew in their place. What syndrome does this person most likely have?


A. Cowden syndrome

B. Cronkhite-Canada

C. Hypohidrotic ectodermal dysplasia

D. Schopf-Schulz-PassargeCorrect Choice

E. Gardner syndrome




                                                    32
Schopf-Schulz-Passarge syndrome is associated with hydrocystomas of the eyelids, hypotrichosis
(near complete loss of hair early in life), hypodontia, nail abnormalities and multiple palmoplantar
eccrine syringofibroadenomas. The other listed syndromes do not fit the description above


115) The presence of natal teeth and pincer nails suggests which disease entity?

A. anhidrotic ectodermal dysplasia

B. pachyonychia congenitaCorrect Choice

C. congenital syphillis

D. thalidomide exposure in utero

E. incontinentia pigmenti
Pachyonychia congentia is an autosomal dominant condition characterized by a constellation of
findings affecting ectodermal structures. These include the presence of natal teeth, steatocystoma
multiplex, follicular hyperkeratosis of the knees, elbows and extensor extremities, eruptive vellus
hair cysts, and oral leukokeratosis which is not pre-malignant. In addition, nail findings include
twenty-nail dystrophy, subungual hyperkeratosis with increase transverse curvature ("pincer nails")
and candidal paronychia. There are two forms of pachyonychia congenital: Type 1 (Jadassohn-
Lewandowsky syndrome) caused by defects in keratin 6a and 16, and Type 2 (Jackson-Lawler type)
caused by defects in keratins 6b and 17.
Anhidrotic ectodermal dysplasia is associated with peg-shaped teeth, hypoanodontia, and a non-
specific nail dystrophy. Likewise, incontientia pigmenti also is characterized by anodontia and peg-
shaped teeth and dystrophic changes of the nail. Finally congenital syphilis is a well-recognized
cause of pegged teeth. Limb deformities are the most serious sequelae of thalidomide exposure in
utero


116) A patient with pseudoherpetic keratitis and a painful PPK would improve with a diet low in:

A. Zinc

B. Biotin

C. Cytosine

D. Tyrosine/PhenylalanineCorrect Choice

E. Glycine
Pseudoherpetic keratitis and a painful PPK describes a patient with Richner-Hanhart syndrome
(tyrosenemia type II). Treatment is with a diet low in tyrosine and phenylalanine. A diet low in
glycine, cytosine, biotin or zinc would not be helpful in this syndrome


117) A patient with multiple facial trichilemmomas is at risk of which of the following cancers?

A. Basal cell carcinoma

B. Cylindroma

C. Breast carcinoma Correct Choice

D. Oral squamous cell carcinoma

E. Acute leukemia
Cowden syndrome (multiple hamartoma syndrome) is an autosomal dominant disorder caused by
mutations in PTEN, a phosphatase that dephosphorylates tyrosine, serine, and threonine. Clinically,
there are numerous facial trichilemmomas, oral papillomas, acral keratotic papules, sclerotic
fibromas, breast fibroadenomas and adenocarcinomas, thyroid adenomas and adenocarcinomas,
and hamartomatous polyps of the gastrointestinal tract




                                                 33
118) Which of the following signs is not a criteria for the diagnosis of Neurofibromatosis type I?

A. Sphenoid dysplasia

B. Axillary freckling

C. Greater than 5 café-au-lait macules

D. Optic gliomas

E. Bilateral vestibular schwannomasCorrect Choice
Bilateral vestibular schwannomas are related to type II neurofibromatosis. The remaining options
are diagnostic criteria for NF-1.


119) Patients with homocystinuria are deficient in:

A. Transglutaminase

B. Lysyl oxidase

C. Cystathionine synthetaseCorrect Choice

D. Alpha galactosidase A

E. Steroid sulfatase
Homocystinuria is an autosomal recessive disease caused by a deficiency of cystathionine
synthetase. Manifestations of this condition may include mental retardation, marfanoid habitus,
malar flush, ectopic lentis, embolic phenomena and leg ulcers


120) A patient presents with bilateral retinal hemangioblastomas and a capillary malformation on
her neck. Which gene mutation is most likely?


A. MFH1

B. PTH/PTHrP type I receptor

C. VEGF receptor-3

D. Endoglin

E. VHLCorrect Choice
These findings are most characteristic of Von Hippel-Lindau syndrome. VHL is a tumor suppressor
gene which is mutated in this syndrome. Other findings include: renal and pancreatic
cysts/carcinoma, pheochromocytoma, and cerebellar/other CNS hemangioblastomas. Endoglin is
defective in Osler-Weber-Rendu, PTH/PTHrP type I receptor in Maffucci syndrome, VEGF receptor-3
in Nonne-Milroy disease (hereditary lymphedema) and MFH1 in lymphedema and ptosis syndrome


121) The gene defect in LEOPARD syndrome is:

A. ATM

B. KIP2

C. PTPN11 Correct Choice

D. PRKAR1A

E. Neurofibromin




                                                 34
PTPN11 gene is mutated in LEOPARD syndrome. This syndrome consists of the complex of
Lentigines, EKG abnormalities, Ocular hypertelorism, Pulmonary stenosis, Abnormal genitalia,
Retardation of growth and Deafness. PRKAR1A is associated with the Carney complex of diseases.
KIP2 is found in 15% of cases of Beckwith-Wiedermann syndrome. ATM is mutated in ataxia
telangectasia syndrome and Neurofibromin in Neurofibromatosis type I.


122) Hereditary Hemorrhagic Telangiectasia syndrome is transmitted in an autosomal dominant
fashion and can have two variants. Type I is linked to defects in HHT1, the endoglin gene. Type II is
linked to defects in HHT2, the ALK1 gene. What feature that differentiates type I from type II
clinically?


A. Type I families have an increase incidence of pulmonary arteriovenous fistulasCorrect Choice

B. None of the answers are correct

C. Type I families have an increased incidence of hepatic arteriovenous malformations

D. Type II families have an increased incidence of pulmonary arteriovenous fistulas

E. Type II families have a decreased incidence of hepatic arteriovenous malformations
There is an increased incidence of pulmonary arteriovenous fistulas in HHT type I. Type II has an
increased incidence of hepatic arteriovenous malformations.


123) Connexin 30 (GJB6 gene) is defective in which of the following syndromes?

A. Vohwinkel syndrome

B. Erythrokeratoderma variabilis

C. Clouston syndromeCorrect Choice

D. KID syndrome

E. Vohwinkel syndrome variant
Clouston syndrome is associated with a defect in Connexin 30 (GJB6 gene). Findings include
palmoplantar keratoderma with transgradiens, dystrophic nails, sparse hair with absent body,
eyelash, eyebrow hair after puberty. KID syndrome and Vohwinkel syndrome are associated with a
defect in Connexin 26 (GJB2). Vohwinkel syndrome variant is associated with a loricrin defect.
Erythrokeratoderma variabilis has mutations in Connexin 31 (GJB3) and 30.3(GJB4).


124) What protein is deficient in the condition shown?

A. Desmoglein 3

B. SPINK5

C. Calcium ATP’ase IIA2

D. Calcium ATP’ase IIC1Correct Choice

E. PEX-7
The picture shown is Hailey-Hailey disease. This is an autosomally dominant condition with a defect
in Calcium ATP’ase IIC1. On H&E stain, an acantholytic “dilapidated brick wall” appearance is seen.
Calcium ATP’ase IIA2 is defective in Darier’s Disease, PEX-7 in autosomal recessive type Conradi-
Hunermann disease, SPINK5 in Netherton’s disease and Desmoglein 3 in pemphigus vulgaris




                                                 35

Más contenido relacionado

La actualidad más candente

ETAS_MCQ_02 immunodermatology
ETAS_MCQ_02 immunodermatologyETAS_MCQ_02 immunodermatology
ETAS_MCQ_02 immunodermatologyDerma202
 
ETAS_MCQ_16 dermatological drugs
ETAS_MCQ_16 dermatological drugsETAS_MCQ_16 dermatological drugs
ETAS_MCQ_16 dermatological drugsDerma202
 
ETAS_MCQ_15 dermatologic and cosmetic surgery
ETAS_MCQ_15 dermatologic and cosmetic surgeryETAS_MCQ_15 dermatologic and cosmetic surgery
ETAS_MCQ_15 dermatologic and cosmetic surgeryDerma202
 
Dermatology MCQ and AAFP.pptx
Dermatology MCQ and AAFP.pptxDermatology MCQ and AAFP.pptx
Dermatology MCQ and AAFP.pptxAbdulaziz Bagasi
 
ETAS_MCQ_05 dermatopathology
ETAS_MCQ_05 dermatopathologyETAS_MCQ_05 dermatopathology
ETAS_MCQ_05 dermatopathologyDerma202
 
Previous year question on lichen planus based on neet pg, usmle, plab and fmg...
Previous year question on lichen planus based on neet pg, usmle, plab and fmg...Previous year question on lichen planus based on neet pg, usmle, plab and fmg...
Previous year question on lichen planus based on neet pg, usmle, plab and fmg...Abhishek Gupta
 
Dermatology board review
Dermatology board reviewDermatology board review
Dermatology board reviewAhmed Amer
 
Epidermopoeisis - development of skin
Epidermopoeisis - development of skin Epidermopoeisis - development of skin
Epidermopoeisis - development of skin Kriti Maheshwari
 
Dermatologic Emergencies in Children
Dermatologic Emergencies in Children Dermatologic Emergencies in Children
Dermatologic Emergencies in Children Fatima Farid
 
Previous year question on pemphigus vulgaris based on neet pg, usmle, plab an...
Previous year question on pemphigus vulgaris based on neet pg, usmle, plab an...Previous year question on pemphigus vulgaris based on neet pg, usmle, plab an...
Previous year question on pemphigus vulgaris based on neet pg, usmle, plab an...Medico Apps
 
UPDATE ON CONTACT DERMATITIS
UPDATE ON CONTACT DERMATITISUPDATE ON CONTACT DERMATITIS
UPDATE ON CONTACT DERMATITISRohit Singh
 
European Evidence-based (S3) Guidelines for the Treatment of Acne
European Evidence-based (S3) Guidelines for the Treatment of AcneEuropean Evidence-based (S3) Guidelines for the Treatment of Acne
European Evidence-based (S3) Guidelines for the Treatment of AcneUtai Sukviwatsirikul
 

La actualidad más candente (20)

ETAS_MCQ_02 immunodermatology
ETAS_MCQ_02 immunodermatologyETAS_MCQ_02 immunodermatology
ETAS_MCQ_02 immunodermatology
 
ETAS_MCQ_16 dermatological drugs
ETAS_MCQ_16 dermatological drugsETAS_MCQ_16 dermatological drugs
ETAS_MCQ_16 dermatological drugs
 
ETAS_MCQ_15 dermatologic and cosmetic surgery
ETAS_MCQ_15 dermatologic and cosmetic surgeryETAS_MCQ_15 dermatologic and cosmetic surgery
ETAS_MCQ_15 dermatologic and cosmetic surgery
 
Derma.
Derma.Derma.
Derma.
 
Dermatology MCQ and AAFP.pptx
Dermatology MCQ and AAFP.pptxDermatology MCQ and AAFP.pptx
Dermatology MCQ and AAFP.pptx
 
Ganyang MCQ Dermatology
Ganyang MCQ DermatologyGanyang MCQ Dermatology
Ganyang MCQ Dermatology
 
ETAS_MCQ_05 dermatopathology
ETAS_MCQ_05 dermatopathologyETAS_MCQ_05 dermatopathology
ETAS_MCQ_05 dermatopathology
 
Previous year question on lichen planus based on neet pg, usmle, plab and fmg...
Previous year question on lichen planus based on neet pg, usmle, plab and fmg...Previous year question on lichen planus based on neet pg, usmle, plab and fmg...
Previous year question on lichen planus based on neet pg, usmle, plab and fmg...
 
Immunologically mediated skin diseases
Immunologically mediated skin diseasesImmunologically mediated skin diseases
Immunologically mediated skin diseases
 
Acne vulgaris
Acne vulgarisAcne vulgaris
Acne vulgaris
 
Dermatology board review
Dermatology board reviewDermatology board review
Dermatology board review
 
Dermatology for MRCP
Dermatology for MRCPDermatology for MRCP
Dermatology for MRCP
 
Acne scar treatment
Acne scar treatmentAcne scar treatment
Acne scar treatment
 
Epidermopoeisis - development of skin
Epidermopoeisis - development of skin Epidermopoeisis - development of skin
Epidermopoeisis - development of skin
 
Dermatologic Emergencies in Children
Dermatologic Emergencies in Children Dermatologic Emergencies in Children
Dermatologic Emergencies in Children
 
Previous year question on pemphigus vulgaris based on neet pg, usmle, plab an...
Previous year question on pemphigus vulgaris based on neet pg, usmle, plab an...Previous year question on pemphigus vulgaris based on neet pg, usmle, plab an...
Previous year question on pemphigus vulgaris based on neet pg, usmle, plab an...
 
Blaschkoid dermatitis
Blaschkoid dermatitisBlaschkoid dermatitis
Blaschkoid dermatitis
 
UPDATE ON CONTACT DERMATITIS
UPDATE ON CONTACT DERMATITISUPDATE ON CONTACT DERMATITIS
UPDATE ON CONTACT DERMATITIS
 
European Evidence-based (S3) Guidelines for the Treatment of Acne
European Evidence-based (S3) Guidelines for the Treatment of AcneEuropean Evidence-based (S3) Guidelines for the Treatment of Acne
European Evidence-based (S3) Guidelines for the Treatment of Acne
 
Stains in dermatology
Stains in dermatologyStains in dermatology
Stains in dermatology
 

Similar a ETAS_MCQ_03 a genodermatoses

ETAS_MCQ_03 b genodermatoses
ETAS_MCQ_03 b genodermatosesETAS_MCQ_03 b genodermatoses
ETAS_MCQ_03 b genodermatosesDerma202
 
MRCP 2 - Pictures.ppt for osce examination board revi3
MRCP 2 - Pictures.ppt for osce examination board revi3MRCP 2 - Pictures.ppt for osce examination board revi3
MRCP 2 - Pictures.ppt for osce examination board revi3kcstincas
 
Chromosomal brekage syndrome
Chromosomal brekage syndromeChromosomal brekage syndrome
Chromosomal brekage syndromeReetika (jmu)
 
Modes of inheritance (part 2)-Dr.Gourav
Modes of inheritance (part 2)-Dr.GouravModes of inheritance (part 2)-Dr.Gourav
Modes of inheritance (part 2)-Dr.GouravGourav Thakre
 
Epidermolysis bullosa complete
Epidermolysis bullosa  completeEpidermolysis bullosa  complete
Epidermolysis bullosa completeDr.vivek ranjan
 
Subacute cutaneous lupus erythematosus associated with lupus nephritis
Subacute cutaneous lupus erythematosus associated with lupus nephritisSubacute cutaneous lupus erythematosus associated with lupus nephritis
Subacute cutaneous lupus erythematosus associated with lupus nephritisApollo Hospitals
 
100 mc qsintropicalmedicine
100 mc qsintropicalmedicine100 mc qsintropicalmedicine
100 mc qsintropicalmedicineElyas Mohammed
 
Approach to childhood poikiloderma.pptx
Approach to childhood poikiloderma.pptxApproach to childhood poikiloderma.pptx
Approach to childhood poikiloderma.pptxDR. MOHNISH SEKAR
 
Ichthyoses and ichthyosiform
Ichthyoses and ichthyosiformIchthyoses and ichthyosiform
Ichthyoses and ichthyosiformKasoka Ksk
 
Genetic disorders pdf
Genetic disorders pdfGenetic disorders pdf
Genetic disorders pdfjrseema
 
NEPHROTIC SYNDROME IN PAEDIATRIC
NEPHROTIC SYNDROME IN PAEDIATRICNEPHROTIC SYNDROME IN PAEDIATRIC
NEPHROTIC SYNDROME IN PAEDIATRICMona Mofti
 
ROLE OF NEUTROPHILS IN HEALTH & DISEASE.pptx
ROLE OF NEUTROPHILS IN HEALTH & DISEASE.pptxROLE OF NEUTROPHILS IN HEALTH & DISEASE.pptx
ROLE OF NEUTROPHILS IN HEALTH & DISEASE.pptxjasmine918783
 
DISEASES OF GENETIC MUTATION, TREATMENTS(2).pptx
DISEASES OF GENETIC MUTATION, TREATMENTS(2).pptxDISEASES OF GENETIC MUTATION, TREATMENTS(2).pptx
DISEASES OF GENETIC MUTATION, TREATMENTS(2).pptxTasiumujahid
 
Extramammary Paget’s disease a review of the literature.pdf
Extramammary Paget’s disease a review of the literature.pdfExtramammary Paget’s disease a review of the literature.pdf
Extramammary Paget’s disease a review of the literature.pdfAnna443020
 
Diamond Blackfan anaemia .pptx
Diamond Blackfan anaemia .pptxDiamond Blackfan anaemia .pptx
Diamond Blackfan anaemia .pptxPreetiMinhas3
 
Hutchinson-Gilford Progeria Syndrome
Hutchinson-Gilford Progeria SyndromeHutchinson-Gilford Progeria Syndrome
Hutchinson-Gilford Progeria SyndromeMohamadAlhes
 
Chromosomal aberrations,downs syndrome-Dr.Gourav
Chromosomal aberrations,downs syndrome-Dr.GouravChromosomal aberrations,downs syndrome-Dr.Gourav
Chromosomal aberrations,downs syndrome-Dr.GouravGourav Thakre
 

Similar a ETAS_MCQ_03 a genodermatoses (20)

ETAS_MCQ_03 b genodermatoses
ETAS_MCQ_03 b genodermatosesETAS_MCQ_03 b genodermatoses
ETAS_MCQ_03 b genodermatoses
 
MRCP 2 - Pictures.ppt for osce examination board revi3
MRCP 2 - Pictures.ppt for osce examination board revi3MRCP 2 - Pictures.ppt for osce examination board revi3
MRCP 2 - Pictures.ppt for osce examination board revi3
 
Chromosomal brekage syndrome
Chromosomal brekage syndromeChromosomal brekage syndrome
Chromosomal brekage syndrome
 
Modes of inheritance (part 2)-Dr.Gourav
Modes of inheritance (part 2)-Dr.GouravModes of inheritance (part 2)-Dr.Gourav
Modes of inheritance (part 2)-Dr.Gourav
 
Genodermatosis
GenodermatosisGenodermatosis
Genodermatosis
 
Epidermolysis bullosa complete
Epidermolysis bullosa  completeEpidermolysis bullosa  complete
Epidermolysis bullosa complete
 
Genodermatoses
GenodermatosesGenodermatoses
Genodermatoses
 
Subacute cutaneous lupus erythematosus associated with lupus nephritis
Subacute cutaneous lupus erythematosus associated with lupus nephritisSubacute cutaneous lupus erythematosus associated with lupus nephritis
Subacute cutaneous lupus erythematosus associated with lupus nephritis
 
Mrcp Part 2 Witten Exam
Mrcp Part 2 Witten ExamMrcp Part 2 Witten Exam
Mrcp Part 2 Witten Exam
 
100 mc qsintropicalmedicine
100 mc qsintropicalmedicine100 mc qsintropicalmedicine
100 mc qsintropicalmedicine
 
Approach to childhood poikiloderma.pptx
Approach to childhood poikiloderma.pptxApproach to childhood poikiloderma.pptx
Approach to childhood poikiloderma.pptx
 
Ichthyoses and ichthyosiform
Ichthyoses and ichthyosiformIchthyoses and ichthyosiform
Ichthyoses and ichthyosiform
 
Genetic disorders pdf
Genetic disorders pdfGenetic disorders pdf
Genetic disorders pdf
 
NEPHROTIC SYNDROME IN PAEDIATRIC
NEPHROTIC SYNDROME IN PAEDIATRICNEPHROTIC SYNDROME IN PAEDIATRIC
NEPHROTIC SYNDROME IN PAEDIATRIC
 
ROLE OF NEUTROPHILS IN HEALTH & DISEASE.pptx
ROLE OF NEUTROPHILS IN HEALTH & DISEASE.pptxROLE OF NEUTROPHILS IN HEALTH & DISEASE.pptx
ROLE OF NEUTROPHILS IN HEALTH & DISEASE.pptx
 
DISEASES OF GENETIC MUTATION, TREATMENTS(2).pptx
DISEASES OF GENETIC MUTATION, TREATMENTS(2).pptxDISEASES OF GENETIC MUTATION, TREATMENTS(2).pptx
DISEASES OF GENETIC MUTATION, TREATMENTS(2).pptx
 
Extramammary Paget’s disease a review of the literature.pdf
Extramammary Paget’s disease a review of the literature.pdfExtramammary Paget’s disease a review of the literature.pdf
Extramammary Paget’s disease a review of the literature.pdf
 
Diamond Blackfan anaemia .pptx
Diamond Blackfan anaemia .pptxDiamond Blackfan anaemia .pptx
Diamond Blackfan anaemia .pptx
 
Hutchinson-Gilford Progeria Syndrome
Hutchinson-Gilford Progeria SyndromeHutchinson-Gilford Progeria Syndrome
Hutchinson-Gilford Progeria Syndrome
 
Chromosomal aberrations,downs syndrome-Dr.Gourav
Chromosomal aberrations,downs syndrome-Dr.GouravChromosomal aberrations,downs syndrome-Dr.Gourav
Chromosomal aberrations,downs syndrome-Dr.Gourav
 

Más de Derma202

Phototherapy treatment protocol
Phototherapy treatment protocolPhototherapy treatment protocol
Phototherapy treatment protocolDerma202
 
Histopathplogical photos
Histopathplogical photosHistopathplogical photos
Histopathplogical photosDerma202
 
Slide study from ETAS
Slide  study from ETASSlide  study from ETAS
Slide study from ETASDerma202
 
Arab board primary exam in dermatology 2012
Arab board primary exam  in dermatology 2012Arab board primary exam  in dermatology 2012
Arab board primary exam in dermatology 2012Derma202
 
Dermatology
DermatologyDermatology
DermatologyDerma202
 
ETAS_MCQ_14 plants and creatures of dermatologic significance
ETAS_MCQ_14 plants and creatures of dermatologic significanceETAS_MCQ_14 plants and creatures of dermatologic significance
ETAS_MCQ_14 plants and creatures of dermatologic significanceDerma202
 
ETAS_MCQ_13 photobiology and photosensitivity disorders
ETAS_MCQ_13 photobiology and photosensitivity disordersETAS_MCQ_13 photobiology and photosensitivity disorders
ETAS_MCQ_13 photobiology and photosensitivity disordersDerma202
 
ETAS_MCQ_11 disorder of hair and nails
ETAS_MCQ_11 disorder of hair and nailsETAS_MCQ_11 disorder of hair and nails
ETAS_MCQ_11 disorder of hair and nailsDerma202
 
Derm handbook for medical students and junior doctors 2010
Derm handbook for medical students and junior doctors 2010Derm handbook for medical students and junior doctors 2010
Derm handbook for medical students and junior doctors 2010Derma202
 

Más de Derma202 (9)

Phototherapy treatment protocol
Phototherapy treatment protocolPhototherapy treatment protocol
Phototherapy treatment protocol
 
Histopathplogical photos
Histopathplogical photosHistopathplogical photos
Histopathplogical photos
 
Slide study from ETAS
Slide  study from ETASSlide  study from ETAS
Slide study from ETAS
 
Arab board primary exam in dermatology 2012
Arab board primary exam  in dermatology 2012Arab board primary exam  in dermatology 2012
Arab board primary exam in dermatology 2012
 
Dermatology
DermatologyDermatology
Dermatology
 
ETAS_MCQ_14 plants and creatures of dermatologic significance
ETAS_MCQ_14 plants and creatures of dermatologic significanceETAS_MCQ_14 plants and creatures of dermatologic significance
ETAS_MCQ_14 plants and creatures of dermatologic significance
 
ETAS_MCQ_13 photobiology and photosensitivity disorders
ETAS_MCQ_13 photobiology and photosensitivity disordersETAS_MCQ_13 photobiology and photosensitivity disorders
ETAS_MCQ_13 photobiology and photosensitivity disorders
 
ETAS_MCQ_11 disorder of hair and nails
ETAS_MCQ_11 disorder of hair and nailsETAS_MCQ_11 disorder of hair and nails
ETAS_MCQ_11 disorder of hair and nails
 
Derm handbook for medical students and junior doctors 2010
Derm handbook for medical students and junior doctors 2010Derm handbook for medical students and junior doctors 2010
Derm handbook for medical students and junior doctors 2010
 

Último

Top Rated Bangalore Call Girls Richmond Circle ⟟ 9332606886 ⟟ Call Me For Ge...
Top Rated Bangalore Call Girls Richmond Circle ⟟  9332606886 ⟟ Call Me For Ge...Top Rated Bangalore Call Girls Richmond Circle ⟟  9332606886 ⟟ Call Me For Ge...
Top Rated Bangalore Call Girls Richmond Circle ⟟ 9332606886 ⟟ Call Me For Ge...narwatsonia7
 
Manyata Tech Park ( Call Girls ) Bangalore ✔ 6297143586 ✔ Hot Model With Sexy...
Manyata Tech Park ( Call Girls ) Bangalore ✔ 6297143586 ✔ Hot Model With Sexy...Manyata Tech Park ( Call Girls ) Bangalore ✔ 6297143586 ✔ Hot Model With Sexy...
Manyata Tech Park ( Call Girls ) Bangalore ✔ 6297143586 ✔ Hot Model With Sexy...vidya singh
 
Best Rate (Guwahati ) Call Girls Guwahati ⟟ 8617370543 ⟟ High Class Call Girl...
Best Rate (Guwahati ) Call Girls Guwahati ⟟ 8617370543 ⟟ High Class Call Girl...Best Rate (Guwahati ) Call Girls Guwahati ⟟ 8617370543 ⟟ High Class Call Girl...
Best Rate (Guwahati ) Call Girls Guwahati ⟟ 8617370543 ⟟ High Class Call Girl...Dipal Arora
 
(👑VVIP ISHAAN ) Russian Call Girls Service Navi Mumbai🖕9920874524🖕Independent...
(👑VVIP ISHAAN ) Russian Call Girls Service Navi Mumbai🖕9920874524🖕Independent...(👑VVIP ISHAAN ) Russian Call Girls Service Navi Mumbai🖕9920874524🖕Independent...
(👑VVIP ISHAAN ) Russian Call Girls Service Navi Mumbai🖕9920874524🖕Independent...Taniya Sharma
 
Call Girls Varanasi Just Call 9907093804 Top Class Call Girl Service Available
Call Girls Varanasi Just Call 9907093804 Top Class Call Girl Service AvailableCall Girls Varanasi Just Call 9907093804 Top Class Call Girl Service Available
Call Girls Varanasi Just Call 9907093804 Top Class Call Girl Service AvailableDipal Arora
 
Call Girls Gwalior Just Call 9907093804 Top Class Call Girl Service Available
Call Girls Gwalior Just Call 9907093804 Top Class Call Girl Service AvailableCall Girls Gwalior Just Call 9907093804 Top Class Call Girl Service Available
Call Girls Gwalior Just Call 9907093804 Top Class Call Girl Service AvailableDipal Arora
 
Call Girls Bareilly Just Call 8250077686 Top Class Call Girl Service Available
Call Girls Bareilly Just Call 8250077686 Top Class Call Girl Service AvailableCall Girls Bareilly Just Call 8250077686 Top Class Call Girl Service Available
Call Girls Bareilly Just Call 8250077686 Top Class Call Girl Service AvailableDipal Arora
 
Call Girls Cuttack Just Call 9907093804 Top Class Call Girl Service Available
Call Girls Cuttack Just Call 9907093804 Top Class Call Girl Service AvailableCall Girls Cuttack Just Call 9907093804 Top Class Call Girl Service Available
Call Girls Cuttack Just Call 9907093804 Top Class Call Girl Service AvailableDipal Arora
 
Night 7k to 12k Navi Mumbai Call Girl Photo 👉 BOOK NOW 9833363713 👈 ♀️ night ...
Night 7k to 12k Navi Mumbai Call Girl Photo 👉 BOOK NOW 9833363713 👈 ♀️ night ...Night 7k to 12k Navi Mumbai Call Girl Photo 👉 BOOK NOW 9833363713 👈 ♀️ night ...
Night 7k to 12k Navi Mumbai Call Girl Photo 👉 BOOK NOW 9833363713 👈 ♀️ night ...aartirawatdelhi
 
Call Girls Ludhiana Just Call 9907093804 Top Class Call Girl Service Available
Call Girls Ludhiana Just Call 9907093804 Top Class Call Girl Service AvailableCall Girls Ludhiana Just Call 9907093804 Top Class Call Girl Service Available
Call Girls Ludhiana Just Call 9907093804 Top Class Call Girl Service AvailableDipal Arora
 
Pondicherry Call Girls Book Now 9630942363 Top Class Pondicherry Escort Servi...
Pondicherry Call Girls Book Now 9630942363 Top Class Pondicherry Escort Servi...Pondicherry Call Girls Book Now 9630942363 Top Class Pondicherry Escort Servi...
Pondicherry Call Girls Book Now 9630942363 Top Class Pondicherry Escort Servi...Genuine Call Girls
 
Call Girls Dehradun Just Call 9907093804 Top Class Call Girl Service Available
Call Girls Dehradun Just Call 9907093804 Top Class Call Girl Service AvailableCall Girls Dehradun Just Call 9907093804 Top Class Call Girl Service Available
Call Girls Dehradun Just Call 9907093804 Top Class Call Girl Service AvailableDipal Arora
 
Call Girls Nagpur Just Call 9907093804 Top Class Call Girl Service Available
Call Girls Nagpur Just Call 9907093804 Top Class Call Girl Service AvailableCall Girls Nagpur Just Call 9907093804 Top Class Call Girl Service Available
Call Girls Nagpur Just Call 9907093804 Top Class Call Girl Service AvailableDipal Arora
 
Top Rated Bangalore Call Girls Ramamurthy Nagar ⟟ 9332606886 ⟟ Call Me For G...
Top Rated Bangalore Call Girls Ramamurthy Nagar ⟟  9332606886 ⟟ Call Me For G...Top Rated Bangalore Call Girls Ramamurthy Nagar ⟟  9332606886 ⟟ Call Me For G...
Top Rated Bangalore Call Girls Ramamurthy Nagar ⟟ 9332606886 ⟟ Call Me For G...narwatsonia7
 
VIP Hyderabad Call Girls Bahadurpally 7877925207 ₹5000 To 25K With AC Room 💚😋
VIP Hyderabad Call Girls Bahadurpally 7877925207 ₹5000 To 25K With AC Room 💚😋VIP Hyderabad Call Girls Bahadurpally 7877925207 ₹5000 To 25K With AC Room 💚😋
VIP Hyderabad Call Girls Bahadurpally 7877925207 ₹5000 To 25K With AC Room 💚😋TANUJA PANDEY
 
Russian Escorts Girls Nehru Place ZINATHI 🔝9711199012 ☪ 24/7 Call Girls Delhi
Russian Escorts Girls  Nehru Place ZINATHI 🔝9711199012 ☪ 24/7 Call Girls DelhiRussian Escorts Girls  Nehru Place ZINATHI 🔝9711199012 ☪ 24/7 Call Girls Delhi
Russian Escorts Girls Nehru Place ZINATHI 🔝9711199012 ☪ 24/7 Call Girls DelhiAlinaDevecerski
 
Call Girls Ooty Just Call 8250077686 Top Class Call Girl Service Available
Call Girls Ooty Just Call 8250077686 Top Class Call Girl Service AvailableCall Girls Ooty Just Call 8250077686 Top Class Call Girl Service Available
Call Girls Ooty Just Call 8250077686 Top Class Call Girl Service AvailableDipal Arora
 
Premium Bangalore Call Girls Jigani Dail 6378878445 Escort Service For Hot Ma...
Premium Bangalore Call Girls Jigani Dail 6378878445 Escort Service For Hot Ma...Premium Bangalore Call Girls Jigani Dail 6378878445 Escort Service For Hot Ma...
Premium Bangalore Call Girls Jigani Dail 6378878445 Escort Service For Hot Ma...tanya dube
 
Top Rated Hyderabad Call Girls Erragadda ⟟ 6297143586 ⟟ Call Me For Genuine ...
Top Rated  Hyderabad Call Girls Erragadda ⟟ 6297143586 ⟟ Call Me For Genuine ...Top Rated  Hyderabad Call Girls Erragadda ⟟ 6297143586 ⟟ Call Me For Genuine ...
Top Rated Hyderabad Call Girls Erragadda ⟟ 6297143586 ⟟ Call Me For Genuine ...chandars293
 
Night 7k to 12k Chennai City Center Call Girls 👉👉 7427069034⭐⭐ 100% Genuine E...
Night 7k to 12k Chennai City Center Call Girls 👉👉 7427069034⭐⭐ 100% Genuine E...Night 7k to 12k Chennai City Center Call Girls 👉👉 7427069034⭐⭐ 100% Genuine E...
Night 7k to 12k Chennai City Center Call Girls 👉👉 7427069034⭐⭐ 100% Genuine E...hotbabesbook
 

Último (20)

Top Rated Bangalore Call Girls Richmond Circle ⟟ 9332606886 ⟟ Call Me For Ge...
Top Rated Bangalore Call Girls Richmond Circle ⟟  9332606886 ⟟ Call Me For Ge...Top Rated Bangalore Call Girls Richmond Circle ⟟  9332606886 ⟟ Call Me For Ge...
Top Rated Bangalore Call Girls Richmond Circle ⟟ 9332606886 ⟟ Call Me For Ge...
 
Manyata Tech Park ( Call Girls ) Bangalore ✔ 6297143586 ✔ Hot Model With Sexy...
Manyata Tech Park ( Call Girls ) Bangalore ✔ 6297143586 ✔ Hot Model With Sexy...Manyata Tech Park ( Call Girls ) Bangalore ✔ 6297143586 ✔ Hot Model With Sexy...
Manyata Tech Park ( Call Girls ) Bangalore ✔ 6297143586 ✔ Hot Model With Sexy...
 
Best Rate (Guwahati ) Call Girls Guwahati ⟟ 8617370543 ⟟ High Class Call Girl...
Best Rate (Guwahati ) Call Girls Guwahati ⟟ 8617370543 ⟟ High Class Call Girl...Best Rate (Guwahati ) Call Girls Guwahati ⟟ 8617370543 ⟟ High Class Call Girl...
Best Rate (Guwahati ) Call Girls Guwahati ⟟ 8617370543 ⟟ High Class Call Girl...
 
(👑VVIP ISHAAN ) Russian Call Girls Service Navi Mumbai🖕9920874524🖕Independent...
(👑VVIP ISHAAN ) Russian Call Girls Service Navi Mumbai🖕9920874524🖕Independent...(👑VVIP ISHAAN ) Russian Call Girls Service Navi Mumbai🖕9920874524🖕Independent...
(👑VVIP ISHAAN ) Russian Call Girls Service Navi Mumbai🖕9920874524🖕Independent...
 
Call Girls Varanasi Just Call 9907093804 Top Class Call Girl Service Available
Call Girls Varanasi Just Call 9907093804 Top Class Call Girl Service AvailableCall Girls Varanasi Just Call 9907093804 Top Class Call Girl Service Available
Call Girls Varanasi Just Call 9907093804 Top Class Call Girl Service Available
 
Call Girls Gwalior Just Call 9907093804 Top Class Call Girl Service Available
Call Girls Gwalior Just Call 9907093804 Top Class Call Girl Service AvailableCall Girls Gwalior Just Call 9907093804 Top Class Call Girl Service Available
Call Girls Gwalior Just Call 9907093804 Top Class Call Girl Service Available
 
Call Girls Bareilly Just Call 8250077686 Top Class Call Girl Service Available
Call Girls Bareilly Just Call 8250077686 Top Class Call Girl Service AvailableCall Girls Bareilly Just Call 8250077686 Top Class Call Girl Service Available
Call Girls Bareilly Just Call 8250077686 Top Class Call Girl Service Available
 
Call Girls Cuttack Just Call 9907093804 Top Class Call Girl Service Available
Call Girls Cuttack Just Call 9907093804 Top Class Call Girl Service AvailableCall Girls Cuttack Just Call 9907093804 Top Class Call Girl Service Available
Call Girls Cuttack Just Call 9907093804 Top Class Call Girl Service Available
 
Night 7k to 12k Navi Mumbai Call Girl Photo 👉 BOOK NOW 9833363713 👈 ♀️ night ...
Night 7k to 12k Navi Mumbai Call Girl Photo 👉 BOOK NOW 9833363713 👈 ♀️ night ...Night 7k to 12k Navi Mumbai Call Girl Photo 👉 BOOK NOW 9833363713 👈 ♀️ night ...
Night 7k to 12k Navi Mumbai Call Girl Photo 👉 BOOK NOW 9833363713 👈 ♀️ night ...
 
Call Girls Ludhiana Just Call 9907093804 Top Class Call Girl Service Available
Call Girls Ludhiana Just Call 9907093804 Top Class Call Girl Service AvailableCall Girls Ludhiana Just Call 9907093804 Top Class Call Girl Service Available
Call Girls Ludhiana Just Call 9907093804 Top Class Call Girl Service Available
 
Pondicherry Call Girls Book Now 9630942363 Top Class Pondicherry Escort Servi...
Pondicherry Call Girls Book Now 9630942363 Top Class Pondicherry Escort Servi...Pondicherry Call Girls Book Now 9630942363 Top Class Pondicherry Escort Servi...
Pondicherry Call Girls Book Now 9630942363 Top Class Pondicherry Escort Servi...
 
Call Girls Dehradun Just Call 9907093804 Top Class Call Girl Service Available
Call Girls Dehradun Just Call 9907093804 Top Class Call Girl Service AvailableCall Girls Dehradun Just Call 9907093804 Top Class Call Girl Service Available
Call Girls Dehradun Just Call 9907093804 Top Class Call Girl Service Available
 
Call Girls Nagpur Just Call 9907093804 Top Class Call Girl Service Available
Call Girls Nagpur Just Call 9907093804 Top Class Call Girl Service AvailableCall Girls Nagpur Just Call 9907093804 Top Class Call Girl Service Available
Call Girls Nagpur Just Call 9907093804 Top Class Call Girl Service Available
 
Top Rated Bangalore Call Girls Ramamurthy Nagar ⟟ 9332606886 ⟟ Call Me For G...
Top Rated Bangalore Call Girls Ramamurthy Nagar ⟟  9332606886 ⟟ Call Me For G...Top Rated Bangalore Call Girls Ramamurthy Nagar ⟟  9332606886 ⟟ Call Me For G...
Top Rated Bangalore Call Girls Ramamurthy Nagar ⟟ 9332606886 ⟟ Call Me For G...
 
VIP Hyderabad Call Girls Bahadurpally 7877925207 ₹5000 To 25K With AC Room 💚😋
VIP Hyderabad Call Girls Bahadurpally 7877925207 ₹5000 To 25K With AC Room 💚😋VIP Hyderabad Call Girls Bahadurpally 7877925207 ₹5000 To 25K With AC Room 💚😋
VIP Hyderabad Call Girls Bahadurpally 7877925207 ₹5000 To 25K With AC Room 💚😋
 
Russian Escorts Girls Nehru Place ZINATHI 🔝9711199012 ☪ 24/7 Call Girls Delhi
Russian Escorts Girls  Nehru Place ZINATHI 🔝9711199012 ☪ 24/7 Call Girls DelhiRussian Escorts Girls  Nehru Place ZINATHI 🔝9711199012 ☪ 24/7 Call Girls Delhi
Russian Escorts Girls Nehru Place ZINATHI 🔝9711199012 ☪ 24/7 Call Girls Delhi
 
Call Girls Ooty Just Call 8250077686 Top Class Call Girl Service Available
Call Girls Ooty Just Call 8250077686 Top Class Call Girl Service AvailableCall Girls Ooty Just Call 8250077686 Top Class Call Girl Service Available
Call Girls Ooty Just Call 8250077686 Top Class Call Girl Service Available
 
Premium Bangalore Call Girls Jigani Dail 6378878445 Escort Service For Hot Ma...
Premium Bangalore Call Girls Jigani Dail 6378878445 Escort Service For Hot Ma...Premium Bangalore Call Girls Jigani Dail 6378878445 Escort Service For Hot Ma...
Premium Bangalore Call Girls Jigani Dail 6378878445 Escort Service For Hot Ma...
 
Top Rated Hyderabad Call Girls Erragadda ⟟ 6297143586 ⟟ Call Me For Genuine ...
Top Rated  Hyderabad Call Girls Erragadda ⟟ 6297143586 ⟟ Call Me For Genuine ...Top Rated  Hyderabad Call Girls Erragadda ⟟ 6297143586 ⟟ Call Me For Genuine ...
Top Rated Hyderabad Call Girls Erragadda ⟟ 6297143586 ⟟ Call Me For Genuine ...
 
Night 7k to 12k Chennai City Center Call Girls 👉👉 7427069034⭐⭐ 100% Genuine E...
Night 7k to 12k Chennai City Center Call Girls 👉👉 7427069034⭐⭐ 100% Genuine E...Night 7k to 12k Chennai City Center Call Girls 👉👉 7427069034⭐⭐ 100% Genuine E...
Night 7k to 12k Chennai City Center Call Girls 👉👉 7427069034⭐⭐ 100% Genuine E...
 

ETAS_MCQ_03 a genodermatoses

  • 1. Genodermatoses 1) Tyrosinase positive albinism (oculocutaneous albinism type 2) is caused by a mutation in which of the following: A. Tyrosinase B. C-kit C. NEMO D. Tyrosinase related protein 1 E. P gene Correct Choice Oculocutaneous albinism (OCA) type 1 (Tyrosinase negative albinism) is caused by mutations in the tyrosinase gene. OCA type 2 (tyrosinase positive albinism) is caused by mutations in the P gene. OCA type 3 is caused by mutations in the tyrosinase related protein 1 gene. C-kit mutations cause piebaldism and NEMO mutations cause incontinentia pigmenti. 2) A patient with a port wine stain covering one enlarged leg likely has which of the following associated symptoms? A. Enchondromas B. Visceromegaly with omphalocele C. Distichiasis D. Lymphatic and deep venous insufficiency Correct Choice E. Bilateral retinal hemangioblastomas Klippel-Trenaunay-Weber syndrome is a sporadic condition characterized by port-wine stains typically covering one lower extremity that is enlarged with underlying lymphatic and deep venous insufficiency. 3) Patients with Hermansky-Pudlak syndrome may experience which of the following systemic complications as a result of their disease? A. Aortic stenosis B. Rectal abscesses C. Arteriovenous malformations D. Gastroesophageal reflux disease E. Pulmomary fibrosisCorrect Choice Patients with Hermansky-Pudlak syndrome are tyrosinase positive albinos. In addition to their pigmentary dilution and increased risks for cutaneous malignancies, patients lack platelet granules leading to impaired platelet aggregation (and therefore a bleeding diathesis). There is also a lysosomal membrane defect which leads to the accumulation of ceroid lipofuscion in macrophages within the lung (leading to pulmonary fibrosis), gastrointestinal tract (leading to granulomatous colitis), and heart (cardiomyopathy). 4) Non-bullous icthyosiform erythroderma is caused by which of the following mutations A. 12R-lipoxygenase gene (ALOX12B) B. lipoxygenase-3 gene (ALOXE3) C. All of these answers are correctCorrect Choice D. transglutaminase-1 gene (TGM1) 1
  • 2. E. Both 12R-lipoxygenase gene (ALOX12B) and lipoxygenase-3 gene (ALOXE3) are correct Non-bullous congenital erythroderma (NCIE)is an autosomal recessive disorder characterized by a collodion baby presentation at birth, and generalized erythroderma with fine white scale, palmoplantar keratoderma, and heat intolerance. NCIE may be caused by mutations in transglutaminase-1 gene (TGM1), the 12R-lipoxygenase gene (ALOX12B), and the lipoxygenase-3 gene (ALOXE3). Mutations in the keratinocyte TGM1 gene interferes with normal cross-linking of structural proteins and the lipid envelope, leading to defective cornification and desquamation. ALOXE3 functions as an epoxy alcohol synthase using the product of ALOX12B as the preferred substrate; either gene can be the site of mutations causing NCIE. 5) In which of the following Genodermatoses would one find cutaneous hyperpigmentation, blue lunulae and Kayser-Fleishcher rings A. Hemochromatosis B. Osteogenesis Imperfecta C. Marfan's Disease D. Gaucher's Disease E. Wilson's DiseaseCorrect Choice In Wilson's disease (Hepatolenticular Degeneration) one will find a vague greenish discoloration of the skin on the face, neck, and gentalia Hyperpigmentation), azure lunulae (sky-blue moons) of the nails, and Kayser-Fleischer rings. This is due to the body retaining excessive amounts of copper 6) Christ-Siemens-Touraine Syndrome is most commonly linked with defects in which of the following genes? A. ectodysplasin (EDA)Correct Choice B. ERCC2 C. NEMO D. None of these options are correct E. ATP7A Ectodysplasin (EDA) on Xq12-q13 is transmitted in an X-linked recessive fashion and is the most common cause of anhidrotic ectodermal dysplasia (Christ-Siemens-Touraine syndrome or hypohidrotic ectodermal dysplasia). NEMO can be linked to this syndrome and is associated with immunodeficiency, but this is a rare association. ERCC2 is associated with trichothiodystrophy and ATP7A with Menkes kinky hair syndrome. They are not associated with anhidrotic ectodermal dysplasia. 7) Cronkhite-Canada is associated with gastrointestinal polyposis, nail atrophy, alopecia, generalized pigmentation of skin and melanotic macules on the fingers. Which of the following describes its inheritance pattern? A. X-linked dominant B. X-linked recessive C. Autosomal recessive D. Autosomal dominant E. SporadicCorrect Choice Cronkhite-Canada is associated with gastrointestinal polyposis, nail atrophy, alopecia, generalized pigmentation of skin and melanotic macules on the fingers. It is inherited in a sporadic manner. 2
  • 3. 8) A biopsy is taken from a patient with multiple facial lesions is shown. They also have oral papillomas and acral keratotic papules. Which of the following genes is most likely involved in the etiology of these lesions? A. CDKN2A B. PTENCorrect Choice C. NCF1 D. NCF2 E. CYBA The biopsy shown is a trichilemmoma. Patients with Cowden syndrome (multiple hamartoma syndrome) will have multiple facial trichilemmomas in addition to the other findings mentioned. Also present are multiple other adenomas and potentially adenocarcinomas, especially of the thyroid and breast. Hamartomatous polyps of the gastrointestinal tract are also present. The other listed genes are not involved in Cowden syndrome. CYBA, NCF1/2 are involved in chroinc granulomatous disease and CDKN2A is associated with familial dysplastic nevi. 9) Ectopia lentis (downward displacement of the lens) is characteristic of: A. Multiple Carboxylase deficiency B. HomocystinuriaCorrect Choice C. Ehlers-Danlos syndrome D. Phenylketonuria E. Marfan syndrome Ectopia lentis (downward displacement) is seen in homocystinuria. Upward displacement is seen in Marfan syndrome. There are no changes in the lens in phenylketonuria or multiple carboxylase deficiency 10) An infant presents with multiple congenital hemangomas in an generalized distribution. What is the most serious associated condition? A. None of the answers are correct B. All of the answers are correct C. Obstructive jaundice D. Congestive Heart FailureCorrect Choice E. Portal hypertension High output congestive heart failure can lead to death in these children. Obstructive jaundice and portal hypertension both occur, but are less likely to cause death. The hemangiomas will undergo spontaneous regression. 11) What medication may exacerbate this autosomally dominant, acnatholytic disorder? A. LithiumCorrect Choice B. Oral contraceptives C. Phenytoin 3
  • 4. D. Corticosteroids E. Anti-malarials Darier's disease is autosomal dominant condition characterized by hyperkeratotic papules coalescing into warty plaques and cobblestoned papules on mucosal surfaces. The cutaneous manifestations may be exacerbated by lithium. 12) Which of the following subtypes of Ehlers-Danlos Syndrome (EDS) is associated with early demise? A. Type 4 (Vascular) Correct Choice B. Type 5 (X-linked) C. Type 1 (Gravis) D. Type 7 (Arthrochalasis multiplex congenita) E. Type 10 (Fibronectin) Ehlers-Danlos Syndrome type 4 (Vascular) is caused by mutations in collagen 3 in some cases. There is a tendency to develop arterial and visceral rupture resulting in early death. 13) An infant presents with poikiloderma on his face, buttocks, arms and legs. He is also noted to have a hypoplastic thumb and no radius. Yearly ophthalmologic examination is indicated because of the infant is at risk for developing: A. Subcapsular lens displacement B. Macular degeneration C. Glaucoma D. Copper deposition E. CataractsCorrect Choice The patient described has Rothmund-Thomson syndrome (or poikiloderma congentiale), an autosomal recessive disease localized to chromosome 8 and believed to be due to a DNA helicase mutation (RECQL4). 40-50% of patients will develop juvenile cataracts before puberty. Other clinical features include alopecia, dystrophic nails, short stature, hypogonadism and dental dysplasia. 14) Retention of primary teeth a dental finding of which of the following conditions? A. Letterer-Siwe disease B. Tuberous sclerosis C. Hypomelanosis of Ito D. Hyper-IgE syndrome Correct Choice E. Jackson Sertoli syndrome Hyper-Immunoglobulin E syndrome is an autosomal dominant condition with impaired regulation of IgE function and deficient neutrophil chemotaxis. There is increased susceptibilty to infections and increased IgE serum levels. Retained primary teeth and lack of development of secondary teeth are characteristic findings. The remaining conditions do not have this as a prominent finding. 15) A patient with a white, spongy overgrowth of the buccal mucosa that has passed in an autosomal dominant fashion is most likely related to a mutation in which of the following? 4
  • 5. A. Keratin 6b/17 B. Keratin 1/10 C. Keratin 6a/16 D. None of these options are correct E. Keratin 4/13Correct Choice This description is most likely a white sponge nevus, an autosomal dominant defect in keratin 4/13. Keratin 1/10 is mutated in epidermolytic hyperkeratosis and Unna-Thost PPK, Keratin 6a/16 in pachyonychia congenita type I, and keratin 6b/17 in pachyonychia congenita type II. 16) Dental enamel pits are seen in which of the following conditions? A. Hypomelanosis of Ito B. Tuberous sclerosisCorrect Choice C. Hyper-IgE syndrome D. Letterer-Siwe disease E. Jackson Sertoli syndrome Tuberous sclerosis is an autosomal dominant condition caused by mutations of the TSC1 (hamartin) or TSC2 (tuberin) genes. These are tumor suppressor genes. Skin findings include hypopigmented macules, connective tissue nevus, facial angiofibromas, periungual fibromas and cafe au lait macules. Dental enamel pits and gingival fibromas are oral findings that are associated with this condition. The remaining conditions do not have dental pits. 17) A 4-year old boy presents with generalized white scale. The mother reports that her son was born with a tight membrane enveloping his body. Peripheral blood smear is within normal limits. What is the most likely diagnosis? A. ichthyosis vulgaris B. lamellar ichthyosis C. neutral lipid storage disease D. netherton syndrome E. Congenital ichthyosiform erythrodermaCorrect Choice The most likely diagnosis is Congenital ichthyosiform erythroderma. In neutral lipid storage disease, the peropheral blood smear would demonstrate lipid vacuoles in leukocytes and monocytes. Lamellar ichthyosis is characterized by plate-like scale in children/adults. Netherton syndrome is characterized by ichthyosis linearis circumflexa. Ichthyosis vulgaris does not typically present with collodian baby 18) Mosaic mutations in PTEN are seen in which of the following conditions? A. Gardner syndrome B. Noonan syndrome C. Incontinentia pigmenti D. Proteus syndrome Correct Choice E. Beckwith-Wiederman syndrome 5
  • 6. Proteus syndrome is a sporadic condition caused by postzygotic mosaic mutations in PTEN. Clinical features include subcutaneous lymphovenous malformations, capillary malformations, lipomas, connective tissue nevi of palms/soles, hemihypertrophy, frontal bossing, hyperostoses of epiphyses & skull (especially external auditory canal), scoliosis, bilateral ovarian cystadenomas, and parotid monomorphic adenomas. 19) Which syndrome is characterized by hyperhidrosis, lack of pain sensation, hypersalivation, and absent fungiform papillae? A. Rubinstein-Taybi syndrome B. Noonan Syndrome C. Turner Syndrome D. Riley-Day Correct Choice E. Cornelia de lange Syndrome Riley-Day syndrome is also known as Familial Dysautonomia. It is an autosomal recessive disorder with the gene defect on the long arm of chromosome 9. Patients have unmyelinated sensory and sympathetic neurons and autonomic dysfunction, leading to hyperhidrosis, decreased corneal sensation and tear flow, hypersalivation, gastroesophageal reflux, decreased deep tendon reflexes, and lack of pain sensation. They also exhibit abnormal histamine skin test. 20) Homocystinuria is caused by a defect in: A. holocarboxylase synthetase B. biotinidase C. phenylalanine hydroxylase D. cystathione beta-synthetaseCorrect Choice E. gp91-phox Cystathione beta-synthetase is defective in homocystinuria, an autosomal recessive conditions characterized by increased homocystine and methionine levels in blood and urine. Other findings include a malar flush, DVTs/emboli, cardiovascular disease, livedo reticularis, leg ulcers, blonde hair/fiar complexion, downward lens dislocation, glaucoma, mental retardation, seizures, psychiatric disorders and a marfanoid body habitus. The other enzymes are not involved in this condition. 21) Hypoplasia of the breast can be seen in which disease? A. anhidrotic ectodermal dysplasiaCorrect Choice B. osteogenesis imperfecta C. congenital syphilis D. Marfan syndrome E. Maffucci syndrome Anhidrotic ectodermal dysplasia is a X-linked recessive disease caused by mutations in ectodysplasin, a member of the tumor necrosis family. Patients may have dry skin with pigmentation periorbitally, hypohidrosis, sparse hair, hypo- anodontia, nail dystrophy, and frontal bossing, and 6
  • 7. saddle nose deformity. In addition to abnormalities of other ectodermally derived structures, the breast and nipple-areolar complex may be absent or hypoplastic. 22) Which radiologic finding is associated in Fanconi's anemia? A. posterior iliac horns B. absent radiiCorrect Choice C. osteopoikilosis D. absent thumb E. osteopathia striata Fanconi's anemia is an inherited disease that primarily affects the bone marrow, resulting in pancytopenia. It is also associated with a broad variety of physical anomalies. Cutaneous findings include cafe-au-lait macules and vitiligo. 23) Birt-Hogg-Dube syndrome is most strongly associated with which of the following malignancies? A. Renal cell carcinoma Correct Choice B. Eccrine syringofibroadenoma C. Trichoepithelial carcinoma D. Medulloblastoma E. Basal cell carcinoma Birt-Hogg-Dube syndrome is characterized by multiple fibrofolliculomas, trichodiscomas, acro- collagenomas, lipomas, and oral fibromas. Patients develop renal cell carcinoma, colon cancer, and medullary thyroid carcinoma. 24) A seven month old infant diagnosed with eczema on her face returns for a diaper-rash follow- up. A one-month trial of topical antifungals has failed to improve the infant’s systems. The part of the physical exam that might prove most useful include: A. Hearing test B. Palpation of abdomen C. Examination of palms and solesCorrect Choice D. Stool samples 7
  • 8. E. Fontanelle examination Acrodermatitis enteropathica presents itself in infancy once breastfeeding has stopped. It can clinically mimic atopic dermatitis, seborrheic dermatitis and candidiasis. Clinical features include scaly red rash around mouth, eyes, and palms, diarrhea, stomatitis, glossitis, alopecia, and failure to thrive. 25) What is the inheritance pattern of chronic granulomatous disease? A. Autosomal dominant B. Sporadic C. X-linked recessiveCorrect Choice D. X-linked dominant E. Autosomal recessive Chronic granulomatous disease is inherited in an x-linked recessive manner. There are mutations present in CYBA (a cytochrome subunit), CYBB, and NCF1 & 2 (neutrophil cytosol factors 1 & 2). 26) Epidermolysis bullosa simplex is caused by blistering in which structure? A. Spinous layer keratinocyte B. Granular layer keratinocyte C. Basal layer keratinocyte Correct Choice D. Sublamina densa E. Lamina densa Epidermolysis bullosa simplex is caused by mutations in keratin 5 and 14 resulting in bullae within basal cell keratinocytes. 27) Hyperextesible skin, gaping wounds, cigarette-paper scars, molluscoid pseudotumors, and calcified subcutaneous nodules are characteristic of which syndrome? A. Marfan syndrome B. Congenital contractural arachnodactyly C. Ehlers-Danlos Syndrome Correct Choice D. Cutis laxa E. Pseudoxanthoma elasticum The most common type of EDS is EDS type I (gravis). Cutaneous features include hyperextensible skin, gaping wounds, cigarette-paper scars, molluscoid pseudotumors, calcified subcutaneous nodules, and bruises. Systemic features include hypermobile joints with dislocation, hernias, mitral valve prolapse, blue sclerae, Gorlin’s sign (tongue reaches nose), and absence of the lingual frenulum 28) Lamellar ichthyosis is caused by mutations in which of the following genes? A. Phytanoyl coenzyme A hydroxylase deficiency B. Arylsulfatase E C. Steroid sulfatase 8
  • 9. D. Transglutaminase Correct Choice E. Fatty aldehyde oxidoreductase Nonbullous congenital ichthyosiform erythroderma and lamellar ichthyosis are caused by mutations in the transglutaminase gene. 29) The coast of Maine cafe au lait macule is a common finding in which of the following syndromes? A. Russell-Silver syndrome B. McCune-Albright syndromeCorrect Choice C. Watson syndrome D. Tuberous Sclerosis E. Neurofibromatosis type I McCune-Albright syndrome is due to a sporadic somatic mutation in Gs subunit of adenylate cyclase. The "coast of Maine" cafe au lait macule is a characteristic finding. Other findings in clude polyostotic fibrous dysplasia and precocious puberty. Tuberous sclerosis, Neurofibromatosis type I, Watson syndrome (pulmonic stenosis and CALM's) and Russell-Silver syndrome (CALMs, short stature, precocious puberty, cryptorchidism and musculoskeletal/craniofacial defects) all have CALMs as a feature, but not the jagged type seen in McCune-Albright syndrome. 30) Enchondromas and chondrosarcomas are most strongly associated with which of the following syndromes? A. Nonne-Milroy disease B. Blue rubber bleb nevus syndrome C. Maffucci syndrome Correct Choice D. Osler-Weber-Rendu syndrome E. Proteus syndrome Maffucci syndrome is a sporadic condition caused by defects in the parathyroid hormone/parathyroid hormone related protein type 1 receptor. There are venous malformations of distal extremities, and benign enchondromas that can degenerate into chondrosarcomas. 31) Epistaxis in early childhood to teens followed by multiple cutaneous and gastrointestinal telangectasias describes which of the following syndromes? A. CREST syndrome B. Maffucci syndrome C. Hereditary Hemorrhagic Telangiectasia syndromeCorrect Choice D. Ataxia telangectasia E. Fabry disease Hereditary Hemorrhagic Telangiectasia syndrome is described above. The first signs in over 50% of cases is epistaxis in childhood to young adulthood. Telangectiasias develop in the 30's and 40's. Other findings include gastrointestinal telangiectasia, hepatic and pulmonary arteriovenous malformations. The other syndromes listed can have cutaneous vascular lesions and should be considered on the differential for hereditary hemorrhagic telangiectasia syndrome 9
  • 10. 32) Which syndrome is characterized by broad thumbs, a large beaked nose, and capillary malformation? A. Rubinstein-TaybiCorrect Choice B. Bloom syndrome C. Proteus syndrome D. Ehlers-Danlos syndrome E. Klinefelter Rubinstein-Taybi syndrome has been associated with a deletion localized to the short arm of chromosome 16. Patients are severely retarded with strabismus, crytorchidism, and congenital heart defects. They have a characteristic beaked nose with nasal septum below alae accompanied by a broad nasal bridge, downslanting palpebral fissures, and broad thumbs and halluces. 33) Which of the following immunoglobulins is commonly decreased in Wiskott-Aldrich syndrome? A. IgE B. IgG C. IgA D. IgMCorrect Choice E. IgD IgM is decreased in WAS. IgA, IgD and IgE levels are all elevated. IgG is not abnormal in WAS. 34) Mutations affecting the VEGF receptor-3 cause which of the following disorders? A. Lymphedema-distichiasis syndrome B. Noonan syndrome C. Lymphedema and ptosis D. Hereditary hemorrhagic telangiectasias E. Hereditary lymphedema (Nonne-Milroy disease) Correct Choice Hereditary lymphedema (Nonne-Milroy disease) is an autosomal dominant condition caused by mutations in the FLT4 gene which encodes for VEGF receptor-3. There is congenital lymphedema and chylous ascites, scrotal swelling, intestinal tract protein loss, persistent bilateral pleural effusion, and hypoproteinemia 35) What is a possible gene defect in this patient with a white forelock? A. tyrosinase B. c-kitCorrect Choice C. P gene D. tyrosinase-related protein 1 E. PAX3 Piebaldism is an autosomal dominant condition caused by a mutation in c-kit which is a mast and stem cell growth factor. This mutation leads to defective melanoblast proliferation and depigmented pathces on the trunk and white forelock. Other features include Hirschprung disease, mental retardation, deafness and cerebellar ataxia. 10
  • 11. 36) Beare-Stevenson cutis gyrata syndrome is linked with mutations in: A. ATP7A B. BSCL2 C. None of these answers are correct D. Fibroblast growth factor receptor 2Correct Choice E. LMNA Beare-Stevenson cutis gyrata syndrome has been linked to mutations in fibroblast growth factor receptor 2. This syndrome is characterized by: craniosynostosis, ciutis gyrata, acanthosis nigricans, anogenital anomalies, skin tags, prominent umbilical stump, furrowed palms and soles. Apert syndrome is also linked to this mutation. BSCL2 is linked to Berardinelli-Seip congenital lipodystrophy, LMNA to Familial partial lipodystrophy and ATP7A to Menkes kinky hair syndrome. 37) Which of the following medications is a teratogen associated with a aplasia cutis congenita? A. Methimazole Correct Choice B. Alcohol C. Lithium D. Warfarin E. Propranolol Aplasia cutis congenita is characterized by well-demarcated erosions at birth that heal with atrophic, alopecic scars. Some cases are caused by medications, with methimazole considered a teratogen particularly associated with this condition. 38) A patient with Cowden syndrome presents with a history of breast carcinoma. What other carcinoma should she be examined/screened for? A. Uterine B. ThyroidCorrect Choice C. Ovarian D. Bone E. Colon Patients with Cowden syndrome are at increase risk for thyroid and breast adenocarcinomas. Ovarian, uterine, colon and bone cancer risks are not elevated in this syndrome. 39) Which ocular finding may be seen in a patient with this skin condition? A. comma-shaped corneal opacities B. congenital hypertrophy of the retinal pigmented epithelium C. retinitis pigmentosa D. angioid streaksCorrect Choice E. pingueculae 11
  • 12. Pseudoxanthoma elasticum is caused by a defect in connective tissue. Angioid streaks develop when a rupture occurs in Bruch;s membrane. 40) Which of the following elastic tissue diseases demonstrates calcified elastic fibers? A. Pseudoxanthoma elasticumCorrect Choice B. Cutis laxa C. Buschke-ollendorf syndrome D. Marfan syndrome E. Anetoderma Pseudoxanthoma elasticum is usually an autosomally recessive inherited condition due to a defective transport protein, ABCC6. The clinical manifestations of the disease arise from fragmented and calcified fibers of the skin, eyes and arteries. Patients may have yellow papules, loose redundant skin, angioid streaks and hemorrhage. Histologically, the hallmark of pseudoxanthoma elasticum is calcified elastic fibers. 41) Junctional epidermolysis bullosa with pyloric atresia is associated with mutations in: A. Laminin 5 B. The alpha-6 subunit of integrin C. The beta-4 subunit of integrin D. Plectin E. Both subunits of integrin can have mutations causing this type of junctional epidermolysis bullosaCorrect Choice Both subunits of integrin can have mutations causing this type of junctional epidermolysis bullosa. Plectin is associated with epidermolysis bullosa simplex with muscular dystrophy. Laminin 5 is mutated in Herlitz and non-Herlitz types of junctional epidermolysis bullosa. 42) Which of the following condition is NOT found in Von-Hippel Lindau syndrome? A. Pheochromocytoma B. Bilateral retinal hemangioblastomas C. Renal cell carcinoma D. Connective tissue neviCorrect Choice E. Cerebellar/CNS hemangioblastomas Von Hippel-Lindau syndrome is characterized by all the options listed except connective tissue nevi. Other findings include pancreatic cysts/carcinoma and cutaneous capillary malformations of the head and neck and polycythemia. 43) Which syndrome is due to a defective secreted mammilian Ly6/uPAR-related protein-1? A. Netherton's syndrome B. Mal de Meleda syndromeCorrect Choice C. Refsum's syndrome D. Haim-Munk syndrome 12
  • 13. E. Sjogren-Larsson syndrome Mal de Meleda, also known as keratoderma palmoplantaris transgrediens, is due to a defect in secreted mammilian Ly6/uPAR-related protein or SLURP-1. 44) You are examining a child with mild albinism, immunodeficiency and silver grey highlights in his hair. You diagnose the child with Chediak-Higashi syndrome. Why are you confident that this isn’t Griscelli syndrome? A. Griscelli syndrome has no changes in hair color B. None of these answers are correct C. All of these answers are correct D. Griscelli syndrome does not have albinism as a feature E. Giant lysosomal granules are present in neutrophils in the blood smearCorrect Choice Chediak-Higashi syndrome and Griscelli syndrome have similar features including silver-grey highlights of hair, immunodeficiency, mild albinism and an accelerated phase of disease. Examining a peripheral blood smear is helpful in distinguishing between these two syndromes. Patients with the LYST defect (a lysosomal storage transport gene) have Chediak-Higashi syndrome and will have giant lysosomal granules visible in white blood cells on a blood smear 45) Adenosine deaminase deficiency is seen in which immunodeficient disease? A. Wiskott-Aldrich syndrome B. Chronic granulomatous disease C. Leiner’s disease D. Job syndrome E. Severe combined immunodeficiency syndromeCorrect Choice Severe combined immunodeficiency is a heterogeneous group of disorders characterized by decreased humoral and cell mediated immunity. Patients may have recurrent infections including cutaneous ones, GVHD (due to in utero cmaternal lymphocytes), sepsis, oral candidiasis, and diarrhea. Implicated genes include the IL-2 receptor (x-linked recessive form) and adenosine deaminase deficiency (autosomal recessive form). 46) Dystrophic epidermolysis bullosa is associated with mutations in collagen VII. Trauma or friction induced blistering in these patients have a plane a splitting in the: A. Stratum spinosum B. None of these answers are correct C. Sublamina densaCorrect Choice D. Stratum basale E. Lamina lucida The split in dystrophic epidermolysis bullosa is found in the sublamina densa, where the collagen VII anchors the epidermis to the anchoring plaques in the dermis. The remaining options are incorrect 47) What is the most likely nail findings in a patient who has this autosomal dominant disease with these keratotic papules and cobblestoning of the oral mucosa? 13
  • 14. A. Red and white longitudinal bandsCorrect Choice B. Koilonychia C. Pincer nails D. Melanonychia E. Half and half nails Darier's disease is an autosomal dominant disorder characterized by greasy hyperkeratotic papules. The papules often coalesce into a warty plaque and have a tendency for secondary viral or bacterial infection. The mutation is in calcium ATPase 2A2. The classic nail finding is red and white longitudinal bands with V-shaped nicking. 48) Low-cystine content in hair and nails may contribute to the phenotype seen in: A. Wilson’s disease B. Bjornstad C. Tay SyndromeCorrect Choice D. Menke’s Kinky Hair syndrome E. Nethertons Tay Syndrome is also known as trichothiodystrophy, or (P)IBIDS: (photosensitivity), icthyosis, brittle hair, intellectual impairment, decreased fertility, and short stature. Hair shaft has a characteristic “tiger tail” appearance under polarized light and the low cystine content in hair and mails is thought to be responsible for the phenotype seen. 49) Bilateral ovarian cystadenomas and parotid monomorphic adenomas are seen in which of the following conditions? A. Beckwith-Wiederman syndrome B. Von-Hippel-Lindau syndrome C. Proteus syndrome Correct Choice D. Cowden syndrome E. Noonan syndrome Proteus syndrome is a sporadic condition caused by postzygotic mosaic mutations in PTEN. Clinical features include subcutaneous lymphovenous malformations, capillary malformations, lipomas, connective tissue nevi of palms/soles, hemihypertrophy, frontal bossing, hyperostoses of epiphyses and skull (esp. external auditory canal), scoliosis, bilateral ovarian cystadenomas, and parotid monomorphic adenomas. 50) Maffucci syndrome is has characteristic venous malformations of the distal extremities and benign endochondromas which can compromise bone strength and lead to chondrosarcomas. The defect causing this believed to be the PTH/PTHrP type I receptor which is inherited in which manner? A. Autosomal recessive B. X-linked recessive C. X-linked dominant D. Autosomal dominant 14
  • 15. E. SporadicCorrect Choice Maffucci syndrome is inherited in a sporadic manner 51) What phenotype results from a low activity of double stranded RNA adenosine deaminase? A. oculocutaneous albinism type 4 B. Piebaldism C. Waardenberg's syndrome type 2 D. Tietz syndrome E. dyschromatosis symmetrica hereditariaCorrect Choice Dyschromatosis symmetrica hereditaria (or acropigmentation symmetrica of Dohi) is an autosomal dominant disease with hypo and hyper pigmented macules and patches on the dorsal hands and feet associated with a low activity of double stranded RNA adenosine deaminase 52) Which of the following is not classically associated with pheochromoctyoma? A. Multiple Endocrine Neoplasia Type IIA B. Neurofibromatosis C. Multiple Endocrine Neoplasia Type IIB D. Cobb SyndromeCorrect Choice E. Von-Hippel-Lindau Syndrome Cobb syndrome is a sporadic disease characterized by cutaneous vascular malformations associated with malformations of the spinal cod. 53) A 17 y/o man presents with facial acne that he would like treated. You notice that he has fine brown scale on his neck and do a complete skin exam. This scale is present on the remainder of his body, sparing his palms, soles and flexural areas. He informs you that his uncles on his mother’s side have similar skin findings. He is not concerned about the skin and would like to proceed with acne treatment only. What other clinical exam should you perform to screen for a potential malignancy to which this patient is at higher risk for acquiring? A. Lung B. Rectal C. Abdominal D. TesticularCorrect Choice E. Lymph node Men with x-linked ichthyosis are at increased risk of testicular cancer and cryptorchidism. A testicular exam is simple to perform and a good screening exam for detecting testicular abnormalities. The remaining exams are not useful as these patients are not at higher risk for other types of cancer 54) Naxos syndrome is characterized by a right sided cardiomyopathy, wooly hair, and keratoderma. The epidermal structure defective in Naxos syndrome is: A. desmoplakin B. plakoglobinCorrect Choice 15
  • 16. C. desmoglein 3 D. desmoglein 1 E. Keratin 1/10 Plakoglobin is an intracellular desmosomal component which binds desmogleins/desmocollins on one side and to desmoplakin on the other. Desmoplakin in turn binds to the keratin intermediate filaments, K1/10 in most cases. Mutation of desmoplakin leads to CarvajaL syndrome, which is associated with a striate palmoplantar keratoderma, woolly hair and Left sided cardiomyopathy. A simple way to remember this is the L in Carvajal cooresponds to the Left sided cardiomyopathy vs. the right sided disease in Naxos disease. 55) What cutaneous manifestation is associated with familial cerebral cavernomas? A. hyperkeratotic cutaneous capillary-venous malformations(HCCVM)Correct Choice B. Verrucous hemangioms C. segmental facial hemangiomas D. Tufted angiomas E. Glomeruloid hemangiomas Familial cerebral cavernomas are due to a defect in the CCM gene which encodes the KRIT-1 protein. These patients often times have hyperkeratotic cutaneous capillary-venous malformations. 56) "Mousy" odor of urine is characteristic of which of the following syndromes? A. Maple Syrup Urine disease B. Hunter disease C. Alkaptonuria D. Hurler disease E. PhenylketonuriaCorrect Choice Phenylketonuria will cause a "mousy" odor in the urine. Patients with Alkaptonuria will have black urine. Maple syrup urine disease will have a sweet odor. Hunter/Hurler diseases are not associated with urinary issues. 57) Which disease can clinically mimic pellagra but is inherited in an autosomal recessive fashion and is due to a defect in the transport of neutral amino acids? A. Fabry B. Wilsons C. Gaucher’s D. Hartnup DiseaseCorrect Choice E. Hemochromatosis The clinical manifestation of Hartnup disease is similar to that of pellagra because the resultant defect in the transport of amino acids leads to low levels of tryptophan. Since tryptophan is required to make nicotinic acid, pts with Hartnup disease manifest the same symptoms as niacin-deficient patients (pellagra). 16
  • 17. 58) A child presents with the hair finding seen in the image in addition to brittle nails, keratosis pilaris, abnormal teeth and cataracts. Which of the following abnormalities is the most likely mutated? A. Keratin hHb1/hHb6Correct Choice B. Keratin 1/10 C. Keratin 2e D. Keratin 6/17 E. Keratin 6/16 Keratin hHb1/hHb6 is defective in monilethrix, which is described above. Keratin 1/10 defects are found in epidermolytic hyperkeratosis, K6/16 in inflamed skin and pachyonychia congenita type I, K6/17 in pachyonychia congenita type II and K2e in Ichythosis bullosa of Siemens 59) Which malignancy is associated with Cowden syndrome? A. Colon cancerCorrect Choice B. Renal cancer C. Lung cancer D. Basal cell cancer E. Ovarian cancer Cowden syndrome is a autosomal dominant syndome with tricholemmomas, oral mucosal papules, acral keratotic papules, thyroid goiter, GI polyps, and fibrocystic breast disease. Malignant associations include breast, thyroid follicular carcinoma, and colon adenocarcinoma. 60) The combination of gastrointestinal polyposis, nail atrophy, alopecia, generalized pigmentation of skin, and melanotic macules of the fingers is characteristic of which of the following syndromes? A. Cronkhite-Canada syndrome Correct Choice B. Bannayan-Riley-Ruvalcaba syndrome C. Cowden syndrome D. Nicolau-Balus syndrome E. Peutz-Jeghers syndrome Cronkhite-Canada syndrome is a sporadic gastrointestinal polyposis syndrome associated with nail atrophy, alopecia, generalized pigmentation of the skin, and melanotic macules on the fingers. 61) Which of the following syndromes is associated with cutis marmorata? A. Netherton’s B. Cornelia de Lange syndromeCorrect Choice C. Hemansky-Pudlak syndrome D. Maffucci syndrome E. Papillon-Lefevre syndrome Cornelia de Lange is also known as Brachmann-de Lange syndrome. Cutaneous manifestations include cutis marmorata, hirsutism, hypoplastic nipples and umbilicus. Patients also have small 17
  • 18. hands and feet. They have characteristic facies which include hirsutism on the forehead, trichomegaly, synophrys, anteverted nostrils, long philtrum, and low-set ears. 62) Lamellar ichthyosis is caused by a defect in transglutaminase 1. It can present as a colloidion baby at birth with subsequent large thick plates of scale especially on flexures, ectropion and eclabium. If two unaffected carrier parents have a child, how likely is their child to have this condition? A. Only male offspring are affected B. None of these answers are correct C. Only female offspring are affected D. 25%Correct Choice E. 75% Lamellar ichthyosis is an autosomal recessive condition. If each parent is a heterozygous carrier, there is a 25% chance that the child will be affected, 50% chance that the child will be a heterozygous carrier and a 25% chance that the child will not be a carrier or affected with lamellar ichthyosis. 63) A 18 yo man presents for evaluation of foot lesions. There are thick hyperkeratotic plaques symmetrically on only the weight bearing plantar surfaces. What test(s) should this patient be referred for? A. Thoracic CT B. Head CT C. EndoscopyCorrect Choice D. Hepatic ultrasound E. Knee films The patient likley has Howel-Evans Syndrome. These patients present with symmetric focal weight bearing PPK in the second decade to adulthood. After the third decade, esophageal carcinoma can occur. These patients should have periodic endoscopic evaluation. 64) To help diagnose trichothiodystrophy, which of the following levels are decreased in hairs of affected individuals? A. Histidine B. Arginine C. CysteineCorrect Choice D. Glycine E. Phenylalanine Cysteine and Methionine levels are decreased in hair and nails of patients with trichothiodystrophy. Other sulfur containing amino acids include: Gluthathione, Taurine, and Homocysteine. Testing hairs for decreased sulfur content is an indirect method of determining this. The other listed amino acids are present in normal levels in the hair and nails of trichothiodystrophy patients 65) Adenosine deaminase deficiency is associated with which of the following disorders? A. Xeroderma pigmentosum 18
  • 19. B. Wiskott Aldrich syndrome C. Gout D. Severe combined immunodeficiencyCorrect Choice E. Job syndrome Adenosine deaminase deficiency is associated with severe combined immunodeficiency. The most common inheritance is x-linked recessive. It is a mixed group of disorders all sharing defects in cell- mediated and humoral immunity. Skin findings include: candidal infections, mucocutaneous, bacterial pyodermas, seborrheic-like dermatitis/lichen planus-like sclerodermatous changes, aplastic thymus and pneumonias. The other listed syndromes are not associated with adenosine deaminase deficiency. 66) Which eye findings would be expected in an individual with this disorder associated with atherosclerosis? A. dendritic corneal ulcerations B. salt and pepper retinitis pigmentosa C. angoid streaksCorrect Choice D. keratoconus E. ectopia lentis Pseudoxanthoma elasticum is an autosomal rescessive or autosomal domminant disease caused by a mutation in ABCC6 (adenosine triphosphate-binding cassette subfamily C member 6). Associated findings include gastric artery hemorrhage, angiod streaks, retinal hemorrhage, atherosclerotic disease, and a possible increased risk of first trimester miscarriage. 67) Which of the following is caused by a mutation in a gene which codes for steroid sulfatase? A. Lamellar icthyosis B. Chediak-Higashi C. X-linked ichthyosisCorrect Choice D. Lesch-nyhan disease E. Fabry disease The mutation in X-linked icthyosis is found in the gene for aryl sulfatase C, a steroid sulfatase. The genetic defect in Fabry disease occurs in alpha-galactosidase A (which hydrolyzes glycolipids and glycoproteins), the defect in lamellar icthyosis codes for transglutaminase 1 and the defect in chediak higashi occurs in a lysosomal transport protein. 68) Meleda is an island off the coast of Croatia. Its inhabitants have an increased frequency of a malodorous transgradiens palmoplantar keratoderma in a stocking-glove distribution. This condition is autosomal recessive with a defect in: A. Cathepsin C B. Keratin 1/9 C. SLURP1 geneCorrect Choice D. TOC gene E. Loricrin 19
  • 20. SLURP1 (Secreted LY6/UPAR-related protein 1) is defective in Mal de Meleda. Keratin 1/9 are defective in Unna-Vorner/Thost palmoplantar keratoderma, an AD diffuse symmetric non- transgradiens PPK. TOC gene is defective in Howel-Evans syndrome, an AD PPK associated with focal, pressure-related, non-transgradiens PPK and esophageal cancer/oral leukoplakia. Loricrin mutations are seen in Vohwinkel syndrome variant and symmetric progressive erythrokeratodermia. Cathepsin C defects are seen in Haim-Munk syndrome (PPK+periodontitis+acroosteolysis+onychogryphosis) and Papillon-Lefevre syndrome (sharply demarcated transgradiens, stocking-glove PPK+periodontitis+dural calcifications and choroids attachments) 69) A patient with thyroid carcinoma and cobblestone-like changes of the oral mucosa will also likely have: A. Syringomas B. Cylindromas C. Trichoepitheliomas D. Fibrofolliculomas E. TricholemmomasCorrect Choice The patient described may have Cowden's syndrome, an autosomal dominant condition caused by a defect in the PTEN tumor suppressor gene. Patients with Cowden's disease are at increased risk for thyroid and breast carcinoma. In addition, they characteristically have multiple hamartomatous polyps of the gastrointestinal tract that are typically benign. Cutaneous features of Cowden's syndrome which may serve as clues to the diagnosis include multiple oral papillomas with a "cobblestone" appearance on the lips, gingival, and buccal mucosa, acral keratotic papules on the dorsal hands and wrists, palmoplantar punctate keratoses and multiple facial tricholemmomas. Patients with this condition need careful malignancy surveillance. 70) What is the inheritance pattern of dermatosis with acantholysis? A. autosomal recessive B. X-linked recessive C. X-linked dominant D. autosomal dominantCorrect Choice E. sporadic Hailey-Hailey, or Familial Benign Pemphigus, is an autosomal dominant genodermatosis which is caused by a mutation in ATP2C1. Vesicles and erythematous plaques develop in the skin folds such as axillae and groin area. 71) A patient with myotonic dystrophy and multiple skin lesions characterized by the pathology image shown most likely has activating mutations in which of the following? A. desmoglein B. desmoplakin C. plakoglobin D. beta-cateninCorrect Choice E. alpha 6-beta 4 integrin Myotonic dystrophy with multiple pilomatricomas is described above. Activating mutations in Beta- catenin are found in this syndrome. The other listed options are desmosomal proteins and are not involved in this syndrome. 20
  • 21. 72) Painful crises and 'whorled' corneal opacities are seen with which of the following enzyme abnormalities? A. alpha-galactosidase ACorrect Choice B. glucocerebrosidase C. iduronate sulfatase D. glucoronidase E. homogentisic acid oxidase Painful crises and whorled corneal opacities are found in Fabry disease which is caused by a defect in alpha-galactosidase A. The remaining conditions do not have these findings 73) A patient with melanoma and a malignant glioma is diagnosed with Li-Fraumeni syndrome. Which of the following tumors occurs most frequently in this disease? A. Breast carcinoma B. Lung carcinoma C. RhabdomyosarcomaCorrect Choice D. Leukemia E. Adrenocortical carcinoma Li-Fraumeni syndrome is a familial tumor syndrome caused by mutations in the tumor suppressor gene p53. They are at risk for a wide range of malignancies with particularly high occurrences of soft tissue sarcomas, breast caner, brain tumors, acute leukemia, and adrenal cortical carcinoma. Soft tissue sarcomas are among the most common reported with this disease. 74) Most common malignancy to develop in a patient with tricholemmomas, acral verrucous papules and cobble-stoning of buccal and gingival mucosa? A. Colon cancer B. Breast cancer C. Melanoma D. Lymphoma E. Thyroid cancerCorrect Choice Cowden's syndrome is an autosomally dominant inherited defect of PTEN. Patient may present with multiple trichilemmoma, hamartomatous tumors of the breast, thyroid and endometrium, acral keratoses and papillomatous papules. Thyroid carcinoma is the most common form malignancy to arise in these patients 75) You are consulted on a patient with possible Nethertons Syndrome. Which location of the body would most likely have hairs demonstrating trichorrhexis invaginata? A. none of these answers are correct B. scalp C. eyebrowCorrect Choice D. all of these answers are correct 21
  • 22. E. eyelash Eyebrow hair is most common site with hairs demonstrating trichorrhexis invaginata. 76) Beckwith-Wiederman syndrome is characterized by which of the following triads? A. Epistaxis, telangictases, and gastrointestinal tract bleeding B. Omphalocele, venous malformations, and ataxia C. Exophthalmos, macroglossia, and gigantism Correct Choice D. Hemangioblastomas, renal cysts and renal cell carcinoma E. Enlarged limb, port wine stain, and deep venous thrombosis Beckwith-Wiederman syndrome is also known as EMG syndrome as it includes exophthalmos, macroglossia, and gigantism. It is usually a sporadic condition but is sometimes caused by autosomal dominant mutations in p57. Clinical features include facial capillary malformations, macroglossia, visceromegaly with omphalocele, and hemihypertrophy associated with tumors (especially Wilm’s tumors). 77) A 16-month old girl presents with patchy alopecia, whorled erythematous scaly eruption, and asymmetric limb shortening. What laboratory or radiologic test may aid in diagnosis? A. Alkaline phosphatase B. Brain MRI C. Bone filmsCorrect Choice D. Chest radiograph E. Complete blood count The patient has Conradi-Hunermann Syndrome. This is a X-linked dominant disorder characterized by ichthyosiform erythroderma in Blaschko's lines in infancy which resolves with follicular atrophoderma, patchy alopecia, short stature, cataracts, scoliosis, assymetric limb shortening. Bone films will demonstrate stippled epiphyses. Ichthyosis and stippled epiphyses resolve after infancy. 78) A patient presents with multiple flesh colored papules on his face. The pathology report comes back as a fibrofolliculoma. He also has multiple lipomas. Which of the following neoplasms must you be concerned about? A. Renal Cell Carcinoma B. Colon carcinoma C. All of the options are correctCorrect Choice D. Medullary thyroid carcinoma E. None of the options are correct Birt-Hogg-Dube syndrome is described above. Renal cell carcinoma is the most common association for internal malignancy, but medullary thyroid and colon carcinomas occur at an increased rate in these individuals also 79) Menke’s kinky hair syndrome is caused by a defect in: A. Gap junction protein B. Mitochondrial gene 22
  • 23. C. Copper Transporting ATPaseCorrect Choice D. DNA helicase E. Proto-oncogene Menke’s kinky hair syndrome is an x-linked recessive disorder caused by a mutation at Xq12 leading to defective intestinal copper transport 80) Retinal hemangioblastomas are found in which syndrome: A. Kasabach-Merritt syndrome B. Klippel-Trenaunay Weber syndrome C. Sturge-Weber syndrome D. Von-Hippel Lindau diseaseCorrect Choice E. Osler-Weber-Rendu disease Von Hippel-Lindau syndrome is an autosomal dominant condition caused by a defect in the VHL tumor suppressor gene. This disease is characterized by retinal hemangioblastomas, often resulting in visual impairment and blindness if left untreated. In addition, many tumors are seen including pheochromocytoma, renal cell carcinoma, and hemangioblastomas of the cerebellum, medulla, and spinal cord. Pancreatic and renal cysts are also a feature of this condition. Finally, polycythemia can occur as a result of erythropoietin production by renal cell carcinoma. Von Hippel-Lindau syndrome is a progressive, universally fatal condition which presents most often in the fourth decade of life 81) Mutations in calcium transporters cause which pair of diseases? A. Chondrodysplasia punctata and CHILD syndrome B. Darier’s disease and Hailey-Hailey disease Correct Choice C. Lamellar ichthyosis and nonbullous congenital ichthyosiform erythroderma D. Refsum syndrome and Sjogren-Larsson syndrome E. Erythrokeratodermia variabilis and progressive symmetric erythrokeratodermia Darier’s disease and Hailey-Hailey disease are caused by mutations in the SERCA calcium ATPase. The former is characterized by hyperkeratotic papules in seborrheic areas, palmar keratoses and pits, red-white longitudinal nail bands, v-shaped distal nail nicks, and cobblestoning of oral and rectal mucosae. The latter is characterized by acantholytic erosions in skin folds 82) Which of the following is a potentially serious complication of the blue rubber bleb nevus syndrome? A. Development of lymphedema B. Gastrointestinal hemorrhage Correct Choice C. Development of chondrosarcomas D. Development of lymphedema E. Development of angiosarcomas Blue rubber bleb nevus syndrome is characterized by multiple tender venous malformations of skin and gastrointestinal tract, which can lead to gastrointestinal bleeding 83) A child with phenylketonuria likely presents with which cutaneous problems? 23
  • 24. A. Alopecia universalis B. Generalized hypopigmentationCorrect Choice C. Leg ulcers D. Blue-gray generalized hyperpigmentation E. Generalized hyperpigmentation Phenylketonuria is an autsomal recessive disorder caused by a mutation on the long arm of chromosome 12. A deficiency of phenylalanine hydroxylase or its cofactor tetrahydrobiopterin leads to accumulation of phenylalanine. Clinical features include generalized hypopigmentation, eczematous dermatitis, sclerodermoid changes, seizures, psychomotor delay, urine with “mousy” odor, mental retardation 84) Patients with Chondrodysplasia punctata can have findings of stippled epiphyses on X-ray examination. Which other x-linked dominant condition can have stippled epiphyses? A. CHILD syndromeCorrect Choice B. Bazex syndrome C. Incontinentia Pigmenti D. Goltz syndrome E. Focal Dermal Hypoplasia All of the syndromes listed have X-linked dominant inheritance. CHILD syndome also has findings of stippled epiphyses. Incontinentia pigmenti is caused by defecdts in the NEMO gene. Findings include peg/conical teeth, eye and CNS defects and alopecia. There are no bone abnormalities. Focal Dermal Hypoplasia, otherwise known as Goltz syndrome has findings of linear atrophy following Blaschko's lines with areas of fat herniation, mucocutaneous papillomas and pits, alopecia, nail dystrophy, tooth abnormalities and osteopathia striata (striations of the long bones). Bazex syndrome is associated with follicular atrophoderma, hypohidrosis, hypotrichosis and multiple basal cell carcinomas. There are no bone abnormalities associated 85) Comma-shaped corneal opacities are characteristic of which type of ichthyosis? A. X-linked ichthyosis Correct Choice B. Nonbullous congenital ichthyosiform erythroderma C. Lamellar ichthyosis D. Ichthyosis vulgaris E. Refsum syndrome X-linked ichthyosis patients have comma-shaped corneal opacities that are asymptomatic yet highly characteristic. 86) Painful crises and 'whorled' corneal opacities are seen with which of the following enzyme abnormalities? A. iduronate sulfatase B. glucoronidase C. glucocerebrosidase D. alpha-galactosidase ACorrect Choice 24
  • 25. E. homogentisic acid oxidase Painful crises and whorled corneal opacities are found in Fabry disease which is caused by a defect in alpha-galactosidase A. The remaining conditions do not have these findings. 87) In chronic granulomatous disease, the diagnosis is made by which of the following tests? A. Assay for fumarate hydratase B. Nitroblue tetrazolium reduction assayCorrect Choice C. Assay for sphingomyelinase D. Skin biopsy E. Assay for glucocerebrosidase Chronic granulomatous disease is diagnosed by the nitroblue tetrazolium reduction assay. The abnormal white blood cells cannot reduce dye due to the inability to produce the respiratory burst. This is needed to kill catalase positive organisms after phagocytosis. Fumarate hydratase is defective in familial multiple cutaneous leiomyomatosis, sphingomyelinase in Niemann-Pick disease and glucocerebrosidase in Gaucher disease 88) Milia, atrophoderma vermiculata and eruptive lesions shown in the pathologic image are characteristic of which of the following syndromes? A. Banayan-Riley-Ruvalcaba B. Gorlin C. Nicolau-BalusCorrect Choice D. Cowden E. Birt-Hogg-Dube Nicolau-Balus syndrome is characterized by eruptive syringomas (shown in the path image), milia an datrophoderma vermiculata. Cowden syndrome is associated with trichilemmomas, Birt-Hogg- Dube with fibrofolliculoma, and Banayan-Riley-Ruvalcaba is not associated with an adnexal neoplasm 89) Which of the following bony defect is found in CHILD syndrome? A. Calcification of falx cerebri B. Osteopoikilosis C. Stippled epiphysesCorrect Choice D. Sphenoid wing dysplasia E. Polyostotic fibrous dysplasia CHILD syndrome is an X-linked dominant syndrome which is lethal in males. It is caused by a peroxisomal biogenesis disorder. It is characterized by unilateral ichthyosiform erythroderma, limb/ visceral hypoplasia, and stippled epiphyses. Stippled epiphyses can also be seen in chondrodysplasia punctata. Polyostotic fibrous dysplasia is found in McCune-Albright syndrome, calcification of falx cerebri in Gorlin's syndrome, osteopoikilosis in seen in Buschke-Ollendorf syndrome 90) Epidermolysis bullosa with muscular dystrophy is caused by mutations in which of the following? 25
  • 26. A. Keratins 5 and 14 B. Loricrin C. Collagen 17 D. Plectin Correct Choice E. Collagen 7 Epidermolysis bullosa with muscular dystrophy is caused by mutations in plectin 91) Which of the following syndromes is associated with hematologic abnormalities? A. Klippel-Trenaunay-Parks-Weber B. Kasabach-Merritt syndromeCorrect Choice C. Bloom’s syndrome D. Blue rubber bleb nevus syndrome E. Sturge Weber disease Kasabach-Merritt syndrome is associated with hematologic abnormalities, such as thrombocytopenia, microangiopathic hemolytic anemia, disseminated intravascular coagulation. The condition develops from platelet-trapping within a large hemangioma, most commonly a kaposiform hemangioendothelioma in the retroperitoneal location. 92) Which of the following is caused by a defect in lysosomal transport: A. Chediak-higashiCorrect Choice B. oculocutaneous albinism II C. Piebaldism D. oculocutaneous albinism I E. Bloom’s syndrome Chediak higashi is an autosomal recessive disorder caused by a mutation in the LYST gene codes for a lysosomal tracking protein. This protein regulates microtubule mediated lysosomal fusion. A defect in this gene leads to giant lysosomal granules seen in neutrophils (leading to defecting phagocytosis and decreased chemotaxis), melanocytes (pigment dilution), and neurons. OCA1 is tyrosinase negative albinism; OCA2 is tyrosinase positive albinism with a mutation in P gene on chromosome 15 93) Which keratins are expressed in the suprabasal palmoplantar epidermis? A. Keratins 4 and 13 B. Keratins 8 and 18 C. Keratins 1 and 9 Correct Choice D. Keratins 5 and 14 26
  • 27. E. Keratins 1 and 10 Keratins are expressed in pairs of acidic and basic keratins and are tissue- and differentiation- specific. Keratins 1 and 9 are expressed in the suprabasal palmoplantar epidermis, keratins 1 and 10 in suprabasal nonpalmoplantar epidermis, keratins 2e and 10 in the granular layer, keratins 4 and 13 in mucosal epithelium, keratins 5 and 14 in the basal layer, and keratins 8 and 18 in simple epithelium. 94) Dermatofibrosis lenticularis disseminata is seen in which of the following conditions? A. Pseudoxanthoma elasticum B. Focal dermal hypoplasia C. Ehlers-Danlos syndrome D. Marfan syndrome E. Buschke-Ollendorf syndrome Correct Choice Buschke-Ollendorf syndrome is an autosomal dominant disorder characterized by dermatofibrosis lenticularis disseminata (cutaneous elastomas distributed symmetrically over the buttocks, trunk and proximal extremities), and osteopoikilosis (round opacities in bones 95) A patient with 20 nail dystrophy, steatocystoma multiplex and natal teeth likely has a mutation in the genes coding for: A. Laminin 5 B. Plakophilin 1 C. Keratins 5 &14 D. Keratins 6b & 17Correct Choice E. Keratins 6 &16 Pachyonychia congenital is an autosomal dominant condition with 20 nail dystrophy. The patient described has Type II (Jackson-Sertole) disease, which includes steatocystoma multiplex, natal teeth, multiple cysts, and micropthalmia, and is caused by mutations in keratins 6b& 17. Type I (Jadassohn-Lewandowsky) also includes focal symmetric PPK, follicular hyperkeratosis, oral leukokeratoses and is caused by mutations in keratins 6 &16. Type III includes the clinical features of type I + corneal leukokeratosis. Mutations in keratins 5&14 represents EB simplex, Laminin 5 mutation is seen in Junctional EB, and plakophilin 1 mutation is seen in ectodermal dysplasia with skin fragility 96) Premalignant leukoplakia of the oral mucosa is associated with: A. Werner Syndrome B. Bloom syndrome C. Rothmund-Thomson syndrome D. Dyskeratosis CongenitaCorrect Choice E. Xeroderma Pigmentosum Dyskeratosis Congenita (also known as Zinsser-Engman-Cole syndrome) is thought to have two modes of inheritance. The more common X-linked disorder is due to a mutation in the Dyskerin gene, while the autosomal dominant form is due to a mutation in TERC, a telomerase RNA component. Clinical features include reticulated gray-brown hyperpigmentation, paloplantar hyperkeratosis, alopecia, onychodystrophy, premalignant leukoplakia of any mucosal surface, and mental retardation 27
  • 28. 97) Which of the following disorders is associated with delayed separation of the umbilical cord? A. Leukocyte adhesion deficiency type 1 (LAD-1)Correct Choice B. Myeloperoxidase deficiency C. X-linked agammaglobulinemia D. Severe combined immunodeficiency disorder(SCID) E. Immunedysregulation, polyendocrinopathy, enteropathy, x-linked (IPEX) LAD-1 manifests as a B2-integrin deficiency and often times presents as at birth with a delayed umbilical cord separation. 98) Mucosal neuromas, pheochromocytoma and medullary thyroid carcinoma in a patient with a marfanoid body habitus is associated with which of the following gene defects? A. RET proto-oncogeneCorrect Choice B. STK11 C. BHD D. Menin E. PTEN The RET proto-oncogene is mutated in Multiple Endocrine Neoplasia type II. Type IIb is described above. Other findings include rare parathyroid abnormalities, megacolon, thickened lips and thick, everted upper eyelids. Menin is associated with MEN type I, PTEN with Cowden disease, BHD with Birt-Hogg-Dube syndrome and STK11 with Peutz-Jeghers syndrome 99) In a patient suspected of having multiple endocrine neoplasia type IIb, which lab test would be appropriate? A. Parathyroid hormone B. Cortisol C. CalcitoninCorrect Choice D. Glucagon E. Calcium Multiple endocrine neoplasia (MEN) syndrome type Iib, also called multiple mucosal neuroma syndrome is an autosomal dominant condition due to a defect in the RET protooncogene on chromosome 10q11.2. This rare condition is associated with mucosal neuromas on the tongue and lips, medullary thyroid carcinoma, pheochromocytoma, and gastrointestinal ganglioneuromatosis. In addition, patients can also present with a marfanoid habitus and facial dysmorphism. Mucosal neuromas can be a dermatologic clue to the underlying diagnosis as these lesions appear during early childhood and present as pink, pedunculated nodules. As the major cause of mortality in these patients is medullary thyroid cancer, which nearly all patients will have by early adulthood, aggressive screening, with serial calcitonin level, and prophylactic thyroidectomy are warranted. 100) What is the gene defect in this condition, which is also called Mendes da Costa syndrome? A. keratin 1 and 10 B. SPINK5 28
  • 29. C. connexin 31Correct Choice D. SLURP-1 E. connexin 26 Mendes da COsta syndrome is also called Erythrokeratoderma Variabilis. It is an autosomal dominantly inherited due to a mutation in connexin 31 or connexin 30.3. It is characterized by transient geographic patches of erythema and hyperkeratotic plaques 101) The hair abnormality shown in the image is characteristic of which of the following diseases? A. TrichothiodystrophyCorrect Choice B. Monilethrix C. Bjornstad syndrome D. All of the options are correct E. Naxos disease The hair abnormality shown is trichoschisis, clean breaks of the hair shaft which occurs in trichothiodystrophy. Also commonly seen is the "tiger-tail" banding pattern of the hair when placed under polarized light. The other options do have hair shaft abnormalities, but not trichoschisis 102) The treatment for acrodermatitis enteropathica is: A. Vitamin B12 supplementation B. Zinc supplementationCorrect Choice C. Phlebotomy D. Iron supplementation E. Vitamin B1 supplementation Acrodermatitis enteropathica is due to a defect in zinc absorption and will respond to zinc supplementation. Iron, Vitamin B1/12 supplementation will not result in improvement in this condition. Findings include periorificial, scalp, and acral dermatitis, scaling, vesicles/bullae, erosions, alopecia, diarrhea and stomatitis 103) In patients with diffuse congenital hemangiomatosis, the most common site for extracutaneous involvement is the : A. Brain B. Thyroid C. LiverCorrect Choice D. Lungs E. Colon Diffuse congenital hemangiomatosis is characterized by multiple hemangiomas with the liver being the most common extracutaneous site, followed by the lungs. Liver hemangioma may be complicated by hepatomegaly, obstructive jaundice, and portal hypertension 104) A deficiency in sialophorin, a surface glycoprotein, is thought to play a role in which genetic disorder? 29
  • 30. A. Bloom Syndrome B. Chediak-higashi syndrome C. Cornelia de Lange syndrome D. Werner syndrome E. Wiskott-aldrich syndromeCorrect Choice Wiskott-Aldrich syndrome is an x-liked recessive disorder localized to Xp11.3. The gene involved codes for WAS, and the protein product has been implicated in lymphocyte and megakaryocyte signal transduction 105) Sphenoid wing dysplasia is seen in: A. Tay Syndrome B. NF-1Correct Choice C. NF-2 D. Mafucci syndrome E. Tuberous sclerosis Sphenoid wing dysplasia is seen in neurofibromatosis type I. Patients with Tay syndrome have short stature, patients with tuberous sclerosis have phalangeal cysts and periosteal thickening, patients with Mafucci syndrome have enchondromas and short stature, and patients with NF-2 do not have any characteristic musculoskeletal findings 106) A patient Buschke-Ollendorff syndrome has osteopoikilosis and which cutaneous finding? A. Epidermal nevi B. Café au lait macules C. Waxy papules along the eyelids D. Port wine stain E. Juvenile elastomaCorrect Choice Buschke-Ollendorf syndrome is an autosomal dominant syndrome associated with increased elastic fiber in the skin. Key features include dermatofibrosis lenticularis disseminata (also called juvenile elastomas) and osteopoikilosis 107) Which of the following is NOT part of the Carney complex? A. Endocrine abnormalities B. Peg or conical teethCorrect Choice C. Pigmented skin lesions D. Cardiac, cutaneous or mammary myxomas E. Primary pigmented nodular adrenocortical disease Peg/conical teeth are not part of the Carney complex. This is found in incontinentia pigmenti and anhidrotic ectodermal dysplasia. The remaining skin findings are part of this complex sometimes known as NAME syndrome. It consists of multiple, diffuse mucocutaneous lentigines, cardiac and subcutaneous myxomas and endocrine abnormalities may be present. Other findings include: testicular tumors, thyroid disease, primary pigmented nodular adrenocortical disease, psammomatous melanotic schwannomas and hormone-secreting pituitary adenomas 30
  • 31. 108) A patient presents with the multiple painful papules on the abdomen with pathology as shown in the image. They report that their father had similar skin lesions. What enzyme defect is the most likely cause? A. Steroid sulfatase B. homogentisic acid oxidase C. glucocerebrosidase D. alpha-galactosidase E. fumarate hydrataseCorrect Choice Fumarate hydratase has been implicated in familial multiple cutaneous leiomyomatosis. Steroid sulfatase is defective in x-linked ichthyosis, alpha-galactosidase in Fabry disease, homogentisic acid oxidase in alkaptonuria and glucocerebrosidase in Gaucher disease. 109) Cutaneous osteomas are seen in which syndrome? A. Carney complex B. Albright hereditary osteodystrophy Correct Choice C. Gaucher’s syndrome D. Waardenburg syndrome E. LEOPARD syndrome Albright hereditary osteodystrophy is caused by mutations in the Gs subunit of adenylate cyclase. There is calcification and ossification due to pseudohypoparathyroidism, absent 4th knuckle, and hypogonadism. 110) What is the genetic defect of this autosomal dominant disorder? A. Calcium ATPase 2C1Correct Choice B. PTPN11 C. SPINK5 D. Calcium ATPase 2A2 E. PTEN Hailey-Hailey is an autosomal dominant disorder that usually affects the intertriginous areas. Clinically, there is erythema and linear fissures of the axilla and groin. On pathology, the characteristic finding is dyskeratosis in a "dilapidated brick wall" pattern. The gene defect responsible is calcium ATPase 2C1 111) Primary pigmented nodular adrenocortical disease and psammomatous melanotic schwannomas are characteristic of which of the following syndromes? A. McCune-Albright syndrome B. Tuberous sclerosis C. Gaucher’s syndrome D. Hypomelanosis of Ito 31
  • 32. E. Carney complex Correct Choice Carney complex is an autosomal dominant disorder caused by mutations in PRKAR1A (protein kinase A regulatory subunit 1-alpha). Key features include cardiac, cutaneous and mammary myxomas, pigmented skin lesions, endocrine abnormalities (pituitary, testicular, thyroid, etc), primary pigmented nodular adrenocortical disease, and psammomatous melanotic schwannomas 112) A triangular-shaped lunula is a characteristic finding in which disease? A. incontinentia pigmenti B. Darier's disease C. dyskeratosis congenita D. nail-patella syndromeCorrect Choice E. epidermal nevus syndrome Nail-patella syndrome, also called hereditary osteo-onychodysplasia is a rare autosomal dominant condition caused by a defect in the LMX1B gene. It is characterized by triangular lunulae, palmoplantar hyperhidrosis, renal dysplasia, glomerulonephritis, and hyperpigmentation of the papillary margin of the iris, an ophthalmologic finding also known as Lester iris. Other nail findings include micronychia with hemionychia, anonychia, and longitudinal fissures. Bony findings include absent or hypoplastic patella, posterior iliac horns, radial head subluxation, thickened scapulae, and scoliosis. Nail findings in Darier's disease include red and white longitudinal bands, subungual hyperkeratosis and V-shaped nicking of the distal nail plate. Dystrophic nails with longitudinal ridges, pterygium, and atrophic or absent nails can be found in dyskeratosis congenita. Dystrophic changes of the nails can be seen in approximately 5-10% of patients with incontinentia pigmenti. 113) A patient with renal cell carcinoma caused by mutations in fumarate hydratase deficiency likely suffers which of the following conditions? A. Multiple endocrine neoplasia B. Birt-Hogg-Dube syndrome C. Von-Hippel-Lindau syndrome D. Familial multiple cutaneous leiomyomatosis Correct Choice E. Cowden syndrome Familial multiple cutaneous leiomyomatosis is an autosomal dominant condition caused by mutations in the fumarate hydratase gene. Clinically, there are multiple cutaneous leiomyomas, uterine leiomyomas and leiomyosarcomas, as well as renal cell carcinomas 114) A patient presents with several light blue cyst-like lesions on the eyelid. They consult their list of problems and bring up plantar hyperkeratosis and dysplastic toenails. On oral exam, you note that they have both upper and lower dentures. The patient relates that after losing their "baby teeth", only 3 teeth grew in their place. What syndrome does this person most likely have? A. Cowden syndrome B. Cronkhite-Canada C. Hypohidrotic ectodermal dysplasia D. Schopf-Schulz-PassargeCorrect Choice E. Gardner syndrome 32
  • 33. Schopf-Schulz-Passarge syndrome is associated with hydrocystomas of the eyelids, hypotrichosis (near complete loss of hair early in life), hypodontia, nail abnormalities and multiple palmoplantar eccrine syringofibroadenomas. The other listed syndromes do not fit the description above 115) The presence of natal teeth and pincer nails suggests which disease entity? A. anhidrotic ectodermal dysplasia B. pachyonychia congenitaCorrect Choice C. congenital syphillis D. thalidomide exposure in utero E. incontinentia pigmenti Pachyonychia congentia is an autosomal dominant condition characterized by a constellation of findings affecting ectodermal structures. These include the presence of natal teeth, steatocystoma multiplex, follicular hyperkeratosis of the knees, elbows and extensor extremities, eruptive vellus hair cysts, and oral leukokeratosis which is not pre-malignant. In addition, nail findings include twenty-nail dystrophy, subungual hyperkeratosis with increase transverse curvature ("pincer nails") and candidal paronychia. There are two forms of pachyonychia congenital: Type 1 (Jadassohn- Lewandowsky syndrome) caused by defects in keratin 6a and 16, and Type 2 (Jackson-Lawler type) caused by defects in keratins 6b and 17. Anhidrotic ectodermal dysplasia is associated with peg-shaped teeth, hypoanodontia, and a non- specific nail dystrophy. Likewise, incontientia pigmenti also is characterized by anodontia and peg- shaped teeth and dystrophic changes of the nail. Finally congenital syphilis is a well-recognized cause of pegged teeth. Limb deformities are the most serious sequelae of thalidomide exposure in utero 116) A patient with pseudoherpetic keratitis and a painful PPK would improve with a diet low in: A. Zinc B. Biotin C. Cytosine D. Tyrosine/PhenylalanineCorrect Choice E. Glycine Pseudoherpetic keratitis and a painful PPK describes a patient with Richner-Hanhart syndrome (tyrosenemia type II). Treatment is with a diet low in tyrosine and phenylalanine. A diet low in glycine, cytosine, biotin or zinc would not be helpful in this syndrome 117) A patient with multiple facial trichilemmomas is at risk of which of the following cancers? A. Basal cell carcinoma B. Cylindroma C. Breast carcinoma Correct Choice D. Oral squamous cell carcinoma E. Acute leukemia Cowden syndrome (multiple hamartoma syndrome) is an autosomal dominant disorder caused by mutations in PTEN, a phosphatase that dephosphorylates tyrosine, serine, and threonine. Clinically, there are numerous facial trichilemmomas, oral papillomas, acral keratotic papules, sclerotic fibromas, breast fibroadenomas and adenocarcinomas, thyroid adenomas and adenocarcinomas, and hamartomatous polyps of the gastrointestinal tract 33
  • 34. 118) Which of the following signs is not a criteria for the diagnosis of Neurofibromatosis type I? A. Sphenoid dysplasia B. Axillary freckling C. Greater than 5 café-au-lait macules D. Optic gliomas E. Bilateral vestibular schwannomasCorrect Choice Bilateral vestibular schwannomas are related to type II neurofibromatosis. The remaining options are diagnostic criteria for NF-1. 119) Patients with homocystinuria are deficient in: A. Transglutaminase B. Lysyl oxidase C. Cystathionine synthetaseCorrect Choice D. Alpha galactosidase A E. Steroid sulfatase Homocystinuria is an autosomal recessive disease caused by a deficiency of cystathionine synthetase. Manifestations of this condition may include mental retardation, marfanoid habitus, malar flush, ectopic lentis, embolic phenomena and leg ulcers 120) A patient presents with bilateral retinal hemangioblastomas and a capillary malformation on her neck. Which gene mutation is most likely? A. MFH1 B. PTH/PTHrP type I receptor C. VEGF receptor-3 D. Endoglin E. VHLCorrect Choice These findings are most characteristic of Von Hippel-Lindau syndrome. VHL is a tumor suppressor gene which is mutated in this syndrome. Other findings include: renal and pancreatic cysts/carcinoma, pheochromocytoma, and cerebellar/other CNS hemangioblastomas. Endoglin is defective in Osler-Weber-Rendu, PTH/PTHrP type I receptor in Maffucci syndrome, VEGF receptor-3 in Nonne-Milroy disease (hereditary lymphedema) and MFH1 in lymphedema and ptosis syndrome 121) The gene defect in LEOPARD syndrome is: A. ATM B. KIP2 C. PTPN11 Correct Choice D. PRKAR1A E. Neurofibromin 34
  • 35. PTPN11 gene is mutated in LEOPARD syndrome. This syndrome consists of the complex of Lentigines, EKG abnormalities, Ocular hypertelorism, Pulmonary stenosis, Abnormal genitalia, Retardation of growth and Deafness. PRKAR1A is associated with the Carney complex of diseases. KIP2 is found in 15% of cases of Beckwith-Wiedermann syndrome. ATM is mutated in ataxia telangectasia syndrome and Neurofibromin in Neurofibromatosis type I. 122) Hereditary Hemorrhagic Telangiectasia syndrome is transmitted in an autosomal dominant fashion and can have two variants. Type I is linked to defects in HHT1, the endoglin gene. Type II is linked to defects in HHT2, the ALK1 gene. What feature that differentiates type I from type II clinically? A. Type I families have an increase incidence of pulmonary arteriovenous fistulasCorrect Choice B. None of the answers are correct C. Type I families have an increased incidence of hepatic arteriovenous malformations D. Type II families have an increased incidence of pulmonary arteriovenous fistulas E. Type II families have a decreased incidence of hepatic arteriovenous malformations There is an increased incidence of pulmonary arteriovenous fistulas in HHT type I. Type II has an increased incidence of hepatic arteriovenous malformations. 123) Connexin 30 (GJB6 gene) is defective in which of the following syndromes? A. Vohwinkel syndrome B. Erythrokeratoderma variabilis C. Clouston syndromeCorrect Choice D. KID syndrome E. Vohwinkel syndrome variant Clouston syndrome is associated with a defect in Connexin 30 (GJB6 gene). Findings include palmoplantar keratoderma with transgradiens, dystrophic nails, sparse hair with absent body, eyelash, eyebrow hair after puberty. KID syndrome and Vohwinkel syndrome are associated with a defect in Connexin 26 (GJB2). Vohwinkel syndrome variant is associated with a loricrin defect. Erythrokeratoderma variabilis has mutations in Connexin 31 (GJB3) and 30.3(GJB4). 124) What protein is deficient in the condition shown? A. Desmoglein 3 B. SPINK5 C. Calcium ATP’ase IIA2 D. Calcium ATP’ase IIC1Correct Choice E. PEX-7 The picture shown is Hailey-Hailey disease. This is an autosomally dominant condition with a defect in Calcium ATP’ase IIC1. On H&E stain, an acantholytic “dilapidated brick wall” appearance is seen. Calcium ATP’ase IIA2 is defective in Darier’s Disease, PEX-7 in autosomal recessive type Conradi- Hunermann disease, SPINK5 in Netherton’s disease and Desmoglein 3 in pemphigus vulgaris 35